You are on page 1of 140

Clinical Sciences

Cell/molecular and membrane


(72)
biology
Clinical anatomy (67)
Clinical biochemistry and metabolism (118)
Clinical physiology (92)
Genetics (107)
Immunology (118)
Statistics (158)

Passing mark range between 63-66%


Year No. of Q
MRCP 1 May 19 100
MRCP 1 Jan 19 100
MRCP 1 Sept 18 100
MRCP 1 May 18 100
MRCP 1 Jan 18 100
MRCP 1 Sept 17 100
MRCP 1 May 17 100
MRCP 1 Jan 17 100
MRCP 1 Sept 16 100
MRCP 1 May 16 100
MRCP 1 Jan 2016 100
MRCP 1 Sept 2015 100
MRCP 1 May 2015 100
MRCP 1 Jan 2015 100

A. H. Murad
‫ﻻ ﺗﻨﺴﻮﻧﺎ ﻣﻦ ﺻﺎﻟﺢ دﻋﺎﺋﻜﻢ‬
‫ﺗﻢ ﺑﺤﻤﺪ ﷲ وﺗﻮﻓﯿﻘﮫ وﻣﻨﮫ‬
‫ﺟﻌﻞ ﷲ ﻋﻤﻠﻨﺎ ﻣﺘﻘﺒﻼ ﺧﺎﻟﺼﺎ ﻟﻮﺟﮭﮫ اﻟﻜﺮﯾﻢ‬
7/29/2019 MyPastest

A 49-year-old man who has recently taken up golf as a hobby comes to the Rheumatology
Clinic complaining of pain in his elbow. He has tenderness over the medial epicondyle of his
right elbow. You proceed to examine the motor function of his hand.

A Abductor pollicis longus

B Adductor pollicis

C Extensor digiti minimi

D Extensor pollicis longus

E Palmaris longus

Explanation 

B Adductor pollicis

The adductor pollicis is innervated by the ulnar nerve and may therefore be affected by ulnar
neuropathy. Clinically, weakness of the adductor pollicis muscle is demonstrated by
Froment's sign, in which a patient is asked to hold a piece of paper between their thumb and
a flat palm as the paper is pulled away. A normal individual can maintain their grip without
difficulty. However, in an affected individual, there is difficulty in maintaining hold on the
object, which is compensated by flexing the thumb.

A Abductor pollicis longus

Abductor pollicis longus is incorrect. Medial epicondylitis, or golfer’s elbow, leads to ulnar
neuropathy in 20% of sufferers, which can then lead to weakness of the muscles innervated
by that nerve. The abductor pollicis longus muscle is innervated by the radial nerve and is
therefore unlikely to be affected.

C Extensor digiti minimi

Extensor digiti minimi is incorrect. The extensor digiti minimi is innervated by the radial nerve.

https://mypastest.pastest.com/Secure/TestMe/Browser/810249 1/2
7/29/2019 MyPastest

D Extensor pollicis longus

Extensor pollicis longus is incorrect. The extensor pollicis longus muscle is also innervated by
the radial nerve.

E Palmaris longus

Palmaris longus is innervated by the median nerve and is less likely than other muscles to be
affected by medial epicondylitis.
37097

Rate this question:

Next Question

Tag Question

Feedback End Session

Difficulty: Difficult

Peer Responses %

Session Progress

Responses Correct: 0

Responses Incorrect: 1

Responses Total: 1

Responses - % Correct: 0%

https://mypastest.pastest.com/Secure/TestMe/Browser/810249 2/2
7/29/2019 MyPastest

A 56-year-old right-handed man presents with left shoulder pain a few days after helping his
daughter move into a new flat. He admits to doing a lot of heavy lifting during the move. He
has no past medical history of note and works as a solicitor. On examination, there is limited
active abduction of the left arm against resistance, with normal passive range of movement,
although he reports pain on passive abduction.

A Acromioclavicular (AC) joint disruption

B Adhesive capsulitis

C Referred neck pain

D Rotator cuff tendonitis

E Impingement syndrome

Explanation 

D Rotator cuff tendonitis

Rotator cuff tendonitis presents most commonly affecting the non-dominant shoulder and is
characterised by defective active abduction and a normal passive range of movements, albeit
often associated with pain. Rotator cuff tears usually follow trauma in younger patients,
although may be atraumatic in the elderly. Treatment in this case would be with NSAIDs and
physiotherapy.

A Acromioclavicular (AC) joint disruption

Acromioclavicular (AC) joint dislocation is incorrect. AC joint dislocation is seen following


traumatic injury (usually a fall onto the affected shoulder). Pain and tenderness are localised
to the AC joint and are accompanied by restriction of both active and passive horizontal
movement of the arm across the body. The clavicle may appear prominent and the scapula
displaced downwards.

B Adhesive capsulitis

https://mypastest.pastest.com/Secure/TestMe/Browser/810249 1/3
7/29/2019 MyPastest

Adhesive capsulitis is incorrect. Adhesive capsulitis (or frozen shoulder) is associated with a
global reduction in passive and active movements and severe pain, and is seen more
commonly in patients with an underlying history of diabetes mellitus.

C Referred neck pain

Referred neck pain is incorrect. Referred neck pain can occasionally be felt as shoulder pain.
In such cases the shoulder should be able to move freely without the pain being aggravated.

E Impingement syndrome

Impingement syndrome is incorrect. Impingement syndrome (also known as supraspinatus


tendinitis and painful arc syndrome) is a condition where the tendons of the rotator cuff
muscles become irritated and inflamed as they pass beneath the acromion. The classical sign
of supraspinatus tendinitis is pain on resisted abduction of between 60 and 120 degrees
when the inflamed tendon presses against the acromium; outside of this range, abduction is
painless. Management may include oral NSAIDs, steroid injections and surgery.
37132

Rate this question:

Next Question

Previous Question Tag Question

Feedback End Session

Difficulty: Average

Peer Responses %

Session Progress

Responses Correct: 0

Responses Incorrect: 2

https://mypastest.pastest.com/Secure/TestMe/Browser/810249 2/3
7/29/2019 MyPastest

Responses Total: 2

Responses - % Correct: 0%

https://mypastest.pastest.com/Secure/TestMe/Browser/810249 3/3
7/29/2019 MyPastest

A 42-year-old motorcyclist comes to the clinic for review. He is suffering from weakness
affecting his left arm. He suffered a fracture of his humerus, which required nail fixation some
4 months earlier. On examination, there is weakness of elbow extension, supination, wrist
extension and thumb abduction. There is sensory loss affecting the dorsal aspect of the
forearm and the lateral portion of the hand, although the little finger is spared.

A Musculocutaneous

B Long thoracic

C Median

D Radial

E Ulnar

Explanation 

D Radial

Radial nerve lesions are well recognised following trauma to the upper arm, including nail
fixation for unstable fractures. Transient radial nerve palsy may also occur due to the
pressure that occurs from sleeping on an outstretched arm. Injury to the radial nerve at
different levels causes different syndromes with varying motor and sensory deficits. Motor
features may include weakness of forearm extension, supination and finger extension.
Sensory features may include loss of sensation to the dorsal aspect of the forearm.

A Musculocutaneous

Musculocutaneous is incorrect. Isolated musculocutaneous nerve injury leads to weakness of


elbow flexion and forearm supination, as well as loss of sensation on the lateral aspect of the
forearm.

B Long thoracic

https://mypastest.pastest.com/Secure/TestMe/Browser/810249 1/3
7/29/2019 MyPastest

Long thoracic is incorrect. Injury to the long thoracic nerve causes weakness of the serratus
anterior muscle. Patients may present with pain, weakness, limitation of shoulder elevation
and scapular winging.

C Median

Median is incorrect. Median nerve lesions lead to weakness of the thenar muscles leading to
an ape-hand deformity. Sensory loss affects the thumb, index and middle fingers, and the
radial aspect of the ring finger.

E Ulnar

Ulnar is incorrect. Injury to the ulnar nerve may also occur at different levels, leading to
different syndromes with varying motor and sensory deficits. Common sites of injury include
the elbow and wrist. Motor features of ulnar nerve injury may lead to claw-hand deformity,
weakness of thumb adduction and weakness of hand flexion. Sensory loss may affect the
medial aspect of the dorsal hand and the medial 1½ digits.
38800

Rate this question:

Next Question

Previous Question Tag Question

Feedback End Session

Difficulty: Average

Peer Responses %

Session Progress

Responses Correct: 0

Responses Incorrect: 3

https://mypastest.pastest.com/Secure/TestMe/Browser/810249 2/3
7/29/2019 MyPastest

Responses Total: 3

Responses - % Correct: 0%

https://mypastest.pastest.com/Secure/TestMe/Browser/810249 3/3
A 50-year-old woman comes to the Emergency Department complaining of sudden loss of
colour vision and blurring affecting her left eye. There is a history of hypertension, for which
she takes amlodipine. Examination reveals a BP of 135/80 mmHg and a regular pulse of
72/min. Her BMI is 23. Fundoscopy reveals a swollen left optic disc, and visual field testing
reveals a left central scotoma.

A Occipital lobe

B Optic chiasm

C Optic nerve

D Optic radiation

E Temporal lobe

Explanation 

C Optic nerve

Central scotoma is an area of depressed vision that corresponds with middle of the visual
field and suggests a lesion in the optic nerve. Common causes include multiple sclerosis,
hypertensive retinal disease and vascular lesions.

A Occipital lobe

Occipital lobe is incorrect. Lesions in the occipital lobe would not be consistent with the
above presentation, but rather cause cortical blindness.

B Optic chiasm

Optic chasm is incorrect. Chiasmal lesions classically produce a bitemporal hemianopia.

D Optic radiation

Optic radiation is incorrect. Lesions in the main optic radiation cause complete homonymous
hemianopia without macular sparing.

E Temporal lobe
Temporal lobe is incorrect. Temporal lobe lesions result in a quadrantanopia.
38853

Rate this question:

Next Question

Previous Question Tag Question

Feedback End Session

Difficulty: Average

Peer Responses %

Session Progress

Responses Correct: 0

Responses Incorrect: 4

Responses Total: 4

Responses - % Correct: 0%
A 72-year-old man with a history of hypertension is admitted to the Emergency Department
by the ambulance team. He collapsed, having got up from the lunch table complaining of
sudden severe headache and neck pain. On arrival in the Emergency Department he is
unconscious with a GCS of 5. His BP is 190/110 mmHg. He is intubated and ventilated and a
CT is performed, which reveals a large pontine haemorrhage.

A Anterior cerebral

B Basilar

C Middle cerebral

D Posterior communicating

E Posterior temporal

Explanation 

B Basilar

The pons is part of the human brainstem and is supplied by small pontine branches of the
basilar artery. These are well-recognised sites of intracerebral haemorrhage related to poorly
controlled hypertension and arterio-venous malformations. Large strokes in this region are
associated with significant mortality.

A Anterior cerebral

Anterior cerebral is incorrect. The anterior cerebral arteries supply the medial portions of the
frontal lobes and the superior aspects of the medial parietal lobes.

C Middle cerebral

Middle cerebral is incorrect. The middle cerebral artery supplies blood to the anterior portion
of the cerebral cortex, including the anterior temporal lobes.

D Posterior communicating

Posterior communicating is incorrect. The posterior communicating artery connects the


posterior cerebral artery to the internal carotid artery.
E Posterior temporal

Posterior temporal is incorrect. The posterior temporal artery supplies blood to the
temporalis muscle.
39980

Rate this question:

Next Question

Previous Question Tag Question

Feedback End Session

Difficulty: Average

Peer Responses %

Session Progress

Responses Correct: 0

Responses Incorrect: 5

Responses Total: 5

Responses - % Correct: 0%
A 74-year-old man with extensive metastases related to prostatic carcinoma comes to the
Emergency Department with dysphagia. Apparently, the problem has increased significantly
over the last few days and he is now unable to drink without choking. Neurological
examination reveals an absent gag reflex, weakness of the neck and a hoarse voice.

A Broca's area

B Geniculate ganglion

C Internal capsule

D Jugular foramen

E Thalamus

Explanation 

D Jugular foramen

The jugular foramen transmits cranial nerves IX, X and XI and the internal jugular vein. Nerve
IX (glossopharyngeal) receives sensory fibres from the tonsils, pharynx, middle ear and
posterior 1/3 of the tongue. It also supplies motor fibres to the stylopharyngeus muscle.
Nerve X (vagus) has both sympathetic and parasympathetic functions. In addition, it supplies
motor fibres to several of the muscles in the palate and pharynx. Finally, nerve XI (accessory)
provides motor control of the sternocleidomastoid and trapezius muscles.

A Broca's area

Broca’s area is incorrect. Broca's area is a region in the frontal lobe of the dominant
hemisphere (usually the left), which is responsible for the production of speech. Lesions in
that area present with expressive aphasia.

B Geniculate ganglion

Geniculate ganglion is incorrect. The geniculate ganglion is one of several ganglia of the head
and neck and receives motor, sensory and parasympathetic fibres of the facial nerve. Lesions
in that region can present with facial nerve palsy and hearing loss.

C Internal capsule
Internal capsule is incorrect. Axons that originate in the motor cortex traverse the internal
capsule. Therefore, lesions in this area present as contralateral hemiplegia.

E Thalamus

Thalamus is incorrect. Lesions affecting the thalamus (most commonly secondary to stroke)
present with contralateral numbness, dysaesthesia, allodynia and tingling.
40044

Rate this question:

Next Question

Previous Question Tag Question

Feedback End Session

Difficulty: Average

Peer Responses %

Session Progress

Responses Correct: 0

Responses Incorrect: 6

Responses Total: 6

Responses - % Correct: 0%
A 24-year-old professional mountain biker comes to the rheumatology clinic for review. He
reports pain in the elbow, particularly at night, and slowly worsening numbness affecting his
little finger and half of his ring finger. There is dull pain over the medial epicondyle of the
elbow on palpation.

A Little finger abduction

B Thumb abduction

C Thumb apposition

D Thumb flexion

E Wrist extension

Explanation 

A Little finger abduction

The position of the elbow pain is consistent with cubital tunnel syndrome, and the sensory
loss fits with ulnar nerve injury. It occurs in patients who are mountain bikers because of
hypertrophy of the forearm muscles. The ulnar nerve supplies the hypothenar eminence and
all of the interossei, hence out of the options given, little finger abduction weakness is the
most likely finding on examination.

B Thumb abduction

Thumb abduction is incorrect. The abductor pollicis brevis muscle is supplied by the median
nerve.

C Thumb apposition

Thumb apposition is incorrect. The opponens pollicis muscle is supplied by the median nerve.

D Thumb flexion

Thumb flexion is incorrect. The flexor pollicis brevis muscle is also supplied by the median
nerve.
E Wrist extension

Wrist extension is incorrect. Weakness of wrist extension is a feature of radial nerve palsy.
48780

Rate this question:

Next Question

Previous Question Tag Question

Feedback End Session

Difficulty: Average

Peer Responses %

Session Progress

Responses Correct: 0

Responses Incorrect: 7

Responses Total: 7

Responses - % Correct: 0%
A motorcyclist involved in a road traffic collision sustained an injury to the brachial plexus on
the right side. He is found to have weakness of right shoulder abduction and forearm flexion,
as well as some sensory loss over the lateral aspect of his upper arm. The right biceps and
brachioradialis reflexes are absent.

A C4,5 root

B C5,6 root

C C6,7 root

D C7,8 root

E C8, T1 root

Explanation 

B C5,6 root

The features are consistent with Erb’s palsy, which is associated with sensory loss over the
lateral aspect of the upper arm (deltoid paralysis), with loss of shoulder abduction and
paralysis of the biceps, brachialis and coracobrachialis. In addition to loss of elbow flexion,
the biceps is also a powerful supinator of the forearm, so the forearm assumes a pronated
position.

A C4,5 root

C4,5 root is incorrect. C4 is responsible for motor supply to the diaphragm, this usually leads
to increased shortness of breath lying flat, which is not reported here.

C C6,7 root

C6,7 root is incorrect. Lesions of C6 and C7 normally result in weakness of elbow and wrist
extension, and pronation of the wrist.

D C7,8 root

C7,8 root is incorrect. Lesions of C7 and C8 result in weakness of wrist flexion and the small
muscles of the hand.
E C8, T1 root

C8, T1 root is incorrect. A T1 lesion produces a claw hand, Klumpke’s palsy. The sympathetic
chain injury results in an associated Horner syndrome, with ptosis of the upper eyelid and
constriction of the pupil (miosis) on the affected side.
48781

Rate this question:

Next Question

Previous Question Tag Question

Feedback End Session

Difficulty: Average

Peer Responses %

Session Progress

Responses Correct: 0

Responses Incorrect: 8

Responses Total: 8

Responses - % Correct: 0%
An 82-year-old woman with atrial fibrillation develops a sudden arterial occlusion of her right
arm owing to a brachial embolism.

A A large, single, intact brachial vein on the medial side of the artery

B Intact circulation distal to the occlusion within the ulnar artery

C Intact sensation in the hand

D Poor capillary return in the fingers

E Return of circulation with systemic heparinisation

Explanation 

D Poor capillary return in the fingers

An occlusion of the brachial artery due to embolism is likely to result in significant symptoms,
including pallor affecting the hand and poor capillary return.

A A large, single, intact brachial vein on the medial side of the artery

A large, single, intact brachial vein on the medial side of the artery is incorrect. The brachial
veins are venae comitantes, accompanying the brachial artery. The artery gives off its
profunda branch near the upper end of the humeral shaft, where it accompanies the radial
nerve.

B Intact circulation distal to the occlusion within the ulnar artery

Intact circulation distal to the occlusion within the ulnar artery is incorrect. The artery
bifurcates into the ulnar and radial arteries; a proximal occlusion would therefore reduce flow
in the ulnar artery.

C Intact sensation in the hand

Intact sensation in the hand is incorrect. Occlusion of the brachial artery will lead to
significant distal loss of sensation.

E Return of circulation with systemic heparinisation


Return of circulation with systemic heparinisation is incorrect. This patient is likely to require
embolectomy.
48782

Rate this question:

Next Question

Previous Question Tag Question

Feedback End Session

Difficulty: Average

Peer Responses %

Session Progress

Responses Correct: 0

Responses Incorrect: 9

Responses Total: 9

Responses - % Correct: 0%
A 38-year-old builder’s labourer sustained a severe fracture of his left elbow, which damaged
the ulnar nerve behind the medial epicondyle of the humerus. A month later, he still has a
total ulnar nerve paralysis.

A Excessive sweating over the ulnar border of the left hand

B Inability to grip a sheet of paper between his fingers when the hand is placed flat on
the table

C Index and middle fingers on the affected side are held in the claw position

D Marked wasting of the thenar eminence

E Sensory loss over the ulnar 3½ digits on the ulnar side of the hand

Explanation 

B Inability to grip a sheet of paper between his fingers when the hand is placed flat on
the table

A test for paralysis of the palmar interossei, supplied by the ulnar nerve, is the inability to
adduct the fingers and thus to be unable to grip a sheet of paper between them.

A Excessive sweating over the ulnar border of the left hand

Excessive sweating over the ulnar border of the left hand is incorrect. Owing to sympathetic
interruption there is absence of sweating in the affected area.

C Index and middle fingers on the affected side are held in the claw position

Index and middle fingers on the affected side are held in the claw position is incorrect. The
fourth and fifth digits are held in the clawed position on the affected side when the nerve is
injured at the wrist. A higher lesion leads to paralysis of long flexors and clawing does not
therefore occur.

D Marked wasting of the thenar eminence

Marked wasting of the thenar eminence is incorrect. The thenar eminence is supplied by the
median nerve.
E Sensory loss over the ulnar 3½ digits on the ulnar side of the hand

Sensory loss over the ulnar 3½ digits on the ulnar side of the hand is incorrect. The ulnar
nerve supplies innervation to the fifth digit and the medial half of the fourth digit, and the
corresponding area of the palm.
48783

Rate this question:

Next Question

Previous Question Tag Question

Feedback End Session

Difficulty: Average

Peer Responses %

Session Progress

Responses Correct: 0

Responses Incorrect: 10

Responses Total: 10

Responses - % Correct: 0%
A 25-year-old man is admitted to the Emergency Department having suffered a severe
laceration to his right thenar eminence after having punched a glass door. An anaesthetist
performs a successful block of the median nerve at the elbow.

A An obvious wrist drop deformity

B Inability to abduct and adduct the fingers

C Inability to abduct the thumb

D Inability to flex the little finger

E The palm of the hand is totally anaesthetised

Explanation 

C Inability to abduct the thumb

Absence of thumb abduction, owing to paralysis of abductor pollicis brevis, is a reliable test
for median nerve paralysis.

A An obvious wrist drop deformity

An obvious wrist drop deformity is incorrect. The radial nerve supplies the wrist extensors;
wrist drop is therefore not seen.

B Inability to abduct and adduct the fingers

Inability to abduct and adduct the fingers is incorrect. The ulnar nerve supplies the interossei,
responsible for this movement.

D Inability to flex the little finger

Inability to flex the little finger is incorrect. The ulnar nerve supplies the muscles responsible
for flexion of the little finger.

E The palm of the hand is totally anaesthetised


The palm of the hand is totally anaesthetised is incorrect. The palm of the hand is supplied by
the ulnar and median nerve.
48784

Rate this question:

Next Question

Previous Question Tag Question

Feedback End Session

Difficulty: Easy

Peer Responses %

Session Progress

Responses Correct: 0

Responses Incorrect: 11

Responses Total: 11

Responses - % Correct: 0%
A 78-year-old man had poliomyelitis as a child, which left him with total paralysis of the left
deltoid muscle.

A Abduction of the shoulder to 60° is likely to be preserved owing to action of the


intact supraspinatus muscle on the left side

B Anaesthesia over the ‘epaulette’ region of the left shoulder

C Detectable weakness in drawing the arm forward and internally rotating the
shoulder when this is compared with the right side

D Drooping of the left shoulder compared with the right side

E The acromion process of the scapula forms the most lateral bony landmark of the
left shoulder

Explanation 

C Detectable weakness in drawing the arm forward and internally rotating the
shoulder when this is compared with the right side

Paralysis of the deltoid muscle causes weakness of shoulder abduction, particularly after 30°
of abduction. The supraspinatus initiates abduction of the shoulder in the first 30° of
movement, therefore this part of the shoulder abduction may be spared (especially in
athletes presenting sports injury to the axillary nerve, for example, who may have
hypertrophied supraspinati).

The deltoid, in addition to being the powerful abductor of the shoulder, assists in flexion and
medial rotation (and extension and lateral rotation) of the shoulder by means of its anterior
and posterior fibres, respectively. Weakness of these movements compared with the normal
side can be detected on careful examination.

A Abduction of the shoulder to 60° is likely to be preserved owing to action of the


intact supraspinatus muscle on the left side

Abduction of the shoulder to 60° is likely to be preserved owing to action of the intact
supraspinatus muscle on the left side is incorrect. Supraspinatus only assists with the first 30°
of abduction.

B Anaesthesia over the ‘epaulette’ region of the left shoulder


Anaesthesia over the ‘epaulette’ region of the left shoulder is incorrect. This area is supplied
by a sensory branch of the axillary nerve. Polio should leave this unaffected.

D Drooping of the left shoulder compared with the right side

Drooping of the left shoulder compared with the right side is incorrect. Scapular elevation is
via the levator scapulae, and the upper fibres of the trapezius.

E The acromion process of the scapula forms the most lateral bony landmark of the
left shoulder

The acromion process of the scapula forms the most lateral bony landmark of the left
shoulder is incorrect. This is more likely to be the head of the humerus where there is deltoid
paralysis.
48785

Rate this question:

Next Question

Previous Question Tag Question

Feedback End Session

Difficulty: Difficult

Peer Responses %

Session Progress

Responses Correct: 0

Responses Incorrect: 12

Responses Total: 12

Responses - % Correct: 0%
An 84-year-old man had his left sciatic nerve completely transected just inferior to the
buttock crease by a piece of shrapnel during military service in his youth.

A Complete anaesthesia below the knee

B Paralysed quadriceps femoris

C Plantar flexed and everted foot

D Spastic paralysis of the lower limb, with increased ankle jerk

E Unimpaired hip abduction

Explanation 

E Unimpaired hip abduction

Hip abduction is effected by the gluteus medius and minimus. These are supplied by the
superior gluteal nerve, and this would be unaffected by transection of the sciatic nerve just
inferior to the buttock crease.

A Complete anaesthesia below the knee

Complete anaesthesia below the knee is incorrect. The saphenous branch of the femoral
nerve would be left intact, meaning normal sensation along the medial side of the anterior
aspect of the leg down to the base of the hallux is maintained.

B Paralysed quadriceps femoris

Paralysed quadriceps femoris is incorrect. This is innervated by the femoral nerve, which
remains intact.

C Plantar flexed and everted foot

Plantar flexed and everted foot is incorrect. The foot in someone with a sciatic transection in
this position would be plantar flexed owing to gravity (foot drop). However, it would be
neither everted (eversion is produced by the peroneus longus and brevis, which are
paralysed) nor inverted, as this is a function of the paralysed long flexors of the foot.
D Spastic paralysis of the lower limb, with increased ankle jerk

Spastic paralysis of the lower limb, with increased ankle jerk is incorrect. Peripheral nerve
injuries result in flaccid paralysis.
48786

Rate this question:

Next Question

Previous Question Tag Question

Feedback End Session

Difficulty: Difficult

Peer Responses %

Session Progress

Responses Correct: 0

Responses Incorrect: 13

Responses Total: 13

Responses - % Correct: 0%
A 75-year-old man has gangrene of the left hallux. There are no pulses to feel below the
rather weak femoral pulse on that side. A duplex scan reveals an occlusion in the superficial
femoral artery.

A Blood flow to the distal foot is provided by collaterals

B Posterior tibial artery flow is still likely to be detectable with a Doppler exam

C Systemic heparin is likely to be effective as an intervention

D The best place to detect flow in the posterior tibial artery is behind the lateral
malleolus

E The femoral vein will be found medial to the artery at the groin

Explanation 

E The femoral vein will be found medial to the artery at the groin

The common femoral artery lies on the lateral side of the vein and divides 3 cm distal to the
inguinal ligament. The superficial femoral artery then becomes the popliteal artery by passing
through the hiatus in the adductor magnus.

A Blood flow to the distal foot is provided by collaterals

Blood flow to the distal foot is provided by collaterals is incorrect. The femoral artery
occlusion is a catastrophic event, and will result in spreading gangrene unless intervention to
relieve the occlusion is attempted.

B Posterior tibial artery flow is still likely to be detectable with a Doppler exam

Posterior tibial artery flow is still likely to be detectable with a Doppler exam is incorrect. The
posterior tibial artery is distal to the femoral artery, and therefore flow is unlikely to be
detectable by Doppler exam.

C Systemic heparin is likely to be effective as an intervention

Systemic heparin is likely to be effective as an intervention is incorrect. Options for


intervention include percutaneous intervention with angioplasty, and locally delivered
thrombolytics.
D The best place to detect flow in the posterior tibial artery is behind the lateral
malleolus

The best place to detect flow in the posterior tibial artery is behind the lateral malleolus is
incorrect. The best position is behind the medial malleolus.
48787

Rate this question:

Next Question

Previous Question Tag Question

Feedback End Session

Difficulty: Average

Peer Responses %

Session Progress

Responses Correct: 0

Responses Incorrect: 14

Responses Total: 14

Responses - % Correct: 0%
A 64-year-old man is admitted with a severe haematemesis. Upper GI endoscopy identifies a
posterior gastric ulcer.

A Gastroduodenal branch of the right gastric artery

B Inferior pancreaticoduodenal artery

C Left gastroepiploic artery

D Oesophageal branch of the left gastric artery

E Splenic artery

Explanation 

E Splenic artery

A posterior gastric ulcer may adhere to, and ulcerate, the splenic artery, as this runs along the
upper border of the pancreas, resulting in a severe haemorrhage.

A Gastroduodenal branch of the right gastric artery

Gastroduodenal branch of the right gastric artery is incorrect. A posterior duodenal ulcer
may erode the gastroduodenal branch of the right gastric artery.

B Inferior pancreaticoduodenal artery

Inferior pancreaticoduodenal artery is incorrect. The inferior pancreaticoduodenal artery


supplies the lower part of the second part of the duodenum, well clear of the site of
ulceration.

C Left gastroepiploic artery

Left gastroepiploic artery is incorrect. A lesser curve gastric ulcer may implicate the left
gastric artery – the gastroepiploic vessels lie along the greater curve of the stomach.

D Oesophageal branch of the left gastric artery


Oesophageal branch of the left gastric artery is incorrect. Oesophageal bleeding occurs via
varices, and it is venous in origin.
48788

Rate this question:

Next Question

Previous Question Tag Question

Feedback End Session

Difficulty: Difficult

Peer Responses %

Session Progress

Responses Correct: 0

Responses Incorrect: 15

Responses Total: 15

Responses - % Correct: 0%
A 50-year-old woman has a suspected right renal artery stenosis. A transfemoral aortogram
has been performed.

A the aorta passes through the diaphragm at the level of the tenth thoracic vertebra

B the catheter will pass through the common femoral artery into first the external iliac
artery and then the aorta at its bifurcation

C the femoral artery at the groin is usually situated halfway between the anterior
superior iliac spine and the pubic tubercle

D the right and left renal arteries will be found in the transpyloric plane at the level of
the first lumbar vertebra

E the right renal artery will have the right ovarian and suprarenal arteries as branches

Explanation 

D the right and left renal arteries will be found in the transpyloric plane at the level of
the first lumbar vertebra

The answer is The right and left renal arteries will be found in the transpyloric plane at the
level of the first lumbar vertebra.

During a transfemoral arteriogram for suspected renal artery stenosis, the catheter is passed
from the femoral artery, through the external iliac artery up into the common iliac artery, and
into the aorta at its bifurcation. From here the catheter is passed up to the level of L1 at
which the right and left renal arteries are located.

A the aorta passes through the diaphragm at the level of the tenth thoracic vertebra

The aorta passes through the diaphragm at the level of the tenth thoracic vertebra is
incorrect. The aorta gives off its renal branches at L1 and passes through the diaphragm at
T12.

B the catheter will pass through the common femoral artery into first the external iliac
artery and then the aorta at its bifurcation

The catheter will pass through the common femoral artery into first the external iliac artery
and then the aorta at its bifurcation is incorrect. The catheter is passed from the femoral
artery, through the external iliac artery up into the common iliac artery, and into the aorta at
its bifurcation.

C the femoral artery at the groin is usually situated halfway between the anterior
superior iliac spine and the pubic tubercle

The femoral artery at the groin is usually situated halfway between the anterior superior iliac
spine and the pubic tubercle is incorrect. The femoral artery at the groin constantly lies
halfway between the anterior superior iliac spine and the midline (the pubic symphysis).

E the right renal artery will have the right ovarian and suprarenal arteries as branches

The right renal artery will have the right ovarian and suprarenal arteries as branches is
incorrect. The aorta gives off the suprarenal and the gonadal (ovarian or testicular) arteries
as separate branches (except for inferior suprarenal artery), respectively, above and below
the origins of the renal arteries on each side.
48789

Rate this question:

Next Question

Previous Question Tag Question

Feedback End Session

Difficulty: Difficult

Peer Responses %

Session Progress

Responses Correct: 0

Responses Incorrect: 16

Responses Total: 16

Responses - % Correct: 0%
A 42-year-old woman with a history of Type 1 diabetes presents to the Emergency
Department, referred with a suspected third nerve palsy.

A A light shone into the affected eye fails to produce constriction of the pupil on the
opposite side

B Constricted pupil on the affected side

C Decreased sweating of the face on the affected side

D Inability to laterally deviate the eye on the affected side

E Ptosis of the upper eyelid on the affected side

Explanation 

E Ptosis of the upper eyelid on the affected side

The oculomotor nerve supplies the levator palpebrae superioris – paralysis of which results in
ptosis (lid lag).

A A light shone into the affected eye fails to produce constriction of the pupil on the
opposite side

A light shone into the affected eye fails to produce constriction of the pupil on the opposite
side is incorrect. This relies on an intact second cranial nerve on the side where the light is
shone, and an intact third nerve on the opposite side. This is therefore normal.

B Constricted pupil on the affected side

Constricted pupil on the affected side is incorrect. The nerve supplies parasympathetic fibres
to the constrictor pupillae, so the pupil widely dilates if this nerve is damaged.

C Decreased sweating of the face on the affected side

Decreased sweating of the face on the affected side is incorrect. The nerve plays no part in
supply to the facial skin.

D Inability to laterally deviate the eye on the affected side


Inability to laterally deviate the eye on the affected side is incorrect. The lateral rectus is
responsible for this movement, and the muscle is controlled by the abducens nerve.
48790

Rate this question:

Next Question

Previous Question Tag Question

Feedback End Session

Difficulty: Average

Peer Responses %

Session Progress

Responses Correct: 0

Responses Incorrect: 17

Responses Total: 17

Responses - % Correct: 0%
A young motorcyclist sustains a skull-base fracture at the level of the middle cranial fossa
that injures his right abducens (VI) nerve.

A The patient is unable to deviate his right eye laterally

B The patient is unable to deviate his right eye medially

C The pupil on the right side is constricted and fails to respond to light

D The right eyelid is numb

E There is ptosis on the right side

Explanation 

A The patient is unable to deviate his right eye laterally

The abducens nerve is responsible for motor supply to the lateral rectus muscle, paralysis of
which leads to a failure of lateral movement of the eye.

B The patient is unable to deviate his right eye medially

The patient is unable to deviate his right eye medially is incorrect. The medial rectus muscle is
supplied by the oculomotor nerve, which also supplies motor fibres to the other ocular
muscles apart from the superior oblique muscle.

C The pupil on the right side is constricted and fails to respond to light

The pupil on the right side is constricted and fails to respond to light is incorrect. The
oculomotor nerve controls pupil constriction. Mononeuritis of the third nerve therefore
results in pupil dilation. Its function is not interrupted here.

D The right eyelid is numb

The right eyelid is numb is incorrect. Sensation to the skin of the face is from the trigeminal
nerve.

E There is ptosis on the right side


There is ptosis on the right side is incorrect. Ptosis is related to an oculomotor lesion rather
than an abducens nerve lesion.
48791

Rate this question:

Next Question

Previous Question Tag Question

Feedback End Session

Difficulty: Easy

Peer Responses %

Session Progress

Responses Correct: 0

Responses Incorrect: 18

Responses Total: 18

Responses - % Correct: 0%
A dental surgeon carries out a block of the inferior alveolar nerve by infiltrating local
anaesthetic at the mandibular foramen.

A Inability of the patient to clench the jaws

B Ineffective block for the incisor teeth

C Numbness of the lower lip on the injected side

D Numbness of the side of the tongue only

E Transient weakness of the facial muscles on the injected side

Explanation 

C Numbness of the lower lip on the injected side

The inferior alveolar nerve is a branch of the mandibular division of the trigeminal nerve (V).
The inferior alveolar nerve traverses the inferior alveolar, or dental, canal of the mandible to
supply all the teeth of that hemi-mandible. All the teeth on that side are therefore
anaesthetised by block of the inferior alveolar nerve. The mental branch of the nerve emerges
through the mental foramen to supply the lower lip, which becomes numb in a successfully
performed block.

A Inability of the patient to clench the jaws

Inability of the patient to clench the jaws is incorrect. The muscles of mastication are not
affected by the nerve block.

B Ineffective block for the incisor teeth

Ineffective block for the incisor teeth is incorrect. The nerve supplies all of the teeth of the
hemi-mandible on the affected side, meaning that the incisor teeth are also anaesthetised.

D Numbness of the side of the tongue only

Numbness of the side of the tongue only is incorrect. Although part of the tongue may
become numb in an alveolar nerve block, the main objective is to numb the teeth and lower
jaw.
E Transient weakness of the facial muscles on the injected side

Transient weakness of the facial muscles on the injected side is incorrect. Facial muscles are
not affected by an inferior alveolar nerve block. Facial nerve blocks are considered for other
procedures such as cataract surgery, not in dental anaesthesia.
48792

Rate this question:

Next Question

Previous Question Tag Question

Feedback End Session

Difficulty: Average

Peer Responses %

Session Progress

Responses Correct: 0

Responses Incorrect: 19

Responses Total: 19

Responses - % Correct: 0%
A patient undergoes a radical parotidectomy for a malignant parotid tumour, at which time it
is found necessary to perform a total division of the left facial (VII) nerve.

A Loss of taste sensation over the anterior two-thirds of the tongue on the left side

B Numbness over the cheek on the left side

C Preservation of left-sided frown in all cases

D Ptosis of the upper eyelid on the left side

E Tendency for food and fluids to collect in the buccal sulcus after meals

Explanation 

E Tendency for food and fluids to collect in the buccal sulcus after meals

This occurs due to paralysis of the buccinator muscle, the action of which is to empty the
buccal sulcus during mastication.

A Loss of taste sensation over the anterior two-thirds of the tongue on the left side

Loss of taste sensation over the anterior two-thirds of the tongue on the left side is incorrect.
The chorda tympani fibres, which transmit taste from the anterior two-thirds of the tongue,
pass from the lingual nerve to the facial nerve just below the skull, and therefore remain
intact in peripheral injuries of the facial nerve.

B Numbness over the cheek on the left side

Numbness over the cheek on the left side is incorrect. Sensation to the face is supplied by the
trigeminal nerve.

C Preservation of left-sided frown in all cases

Preservation of left-sided frown in all cases is incorrect. The facial nerve supplies all the
muscles needed for facial expression including the occipitofrontalis, which wrinkles the
forehead.

D Ptosis of the upper eyelid on the left side


Ptosis of the upper eyelid on the left side is incorrect. The levator palpebrae superioris is
supplied by the oculomotor nerve, so patients with facial nerve transection can still raise their
upper lid.
48793

Rate this question:

Next Question

Previous Question Tag Question

Feedback End Session

Difficulty: Difficult

Peer Responses %

Session Progress

Responses Correct: 0

Responses Incorrect: 20

Responses Total: 20

Responses - % Correct: 0%
A patient undergoes excision of the left submandibular salivary gland for sialectasia.
Unfortunately, his hypoglossal (XII) nerve on that side is damaged.

A All the intrinsic muscles of the left side of the tongue are paralysed

B On protruding the tongue, it deviates towards the right

C The genioglossus muscle is spared

D The uvula deviates towards the left

E There is numbness of the posterior one-third of the tongue

Explanation 

A All the intrinsic muscles of the left side of the tongue are paralysed

The hypoglossal nerve supplies all the muscles of the tongue but none of the palate. Paralysis
then leads to deviation of the tongue to the affected side (the left).

B On protruding the tongue, it deviates towards the right

On protruding the tongue, it deviates towards the right is incorrect. Deviation occurs towards
the paralysed side (i.e. the left).

C The genioglossus muscle is spared

The genioglossus muscle is spared is incorrect. All of the muscles of the tongue on the
damaged side are affected. The genioglossus muscle protrudes the tongue; when it is
paralysed, the muscle on the opposite side is unaffected and deviates the tongue towards the
affected side.

D The uvula deviates towards the left

The uvula deviates towards the left is incorrect. Muscles of the palate are not affected by
hypoglossal nerve injury.

E There is numbness of the posterior one-third of the tongue


There is numbness of the posterior one-third of the tongue is incorrect. The hypoglossal
nerve has no sensory component.
48794

Rate this question:

Next Question

Previous Question Tag Question

Feedback End Session

Difficulty: Average

Peer Responses %

Session Progress

Responses Correct: 0

Responses Incorrect: 21

Responses Total: 21

Responses - % Correct: 0%
A patient has her inferior laryngeal nerve inadvertently divided during a partial
thyroidectomy.

A All the laryngeal muscles are paralysed on the affected side, apart from the
posterior cricoarytenoid muscle

B All the laryngeal muscles on the affected side are paralysed

C At laryngoscopy, the affected cord is seen to lie paralysed in the midline

D The larynx is anaesthetised inferior to the vocal cord on the affected side

E The larynx is totally anaesthetised on the affected side

Explanation 

D The larynx is anaesthetised inferior to the vocal cord on the affected side

The recurrent laryngeal nerve supplies sensory fibres to the larynx inferior to the vocal cords.
This leads to loss of sensation inferior to the vocal cord, in this case on the side where the
nerve was divided.

A All the laryngeal muscles are paralysed on the affected side, apart from the
posterior cricoarytenoid muscle

All the laryngeal muscles are paralysed on the affected side, apart from the posterior
cricoarytenoid muscle is incorrect. The nerve supplies all the muscles of the larynx apart from
the cricothyroid muscle, which is supplied by the superior laryngeal branch of the vagus
nerve.

B All the laryngeal muscles on the affected side are paralysed

All the laryngeal muscles on the affected side are paralysed is incorrect. The cricothyroid
muscle is supplied by the superior laryngeal nerve.

C At laryngoscopy, the affected cord is seen to lie paralysed in the midline

At laryngoscopy, the affected cord is seen to lie paralysed in the midline is incorrect. The
paralysed cord is seen to lie in the ‘paralytic’ position, slightly abducted from the midline, and
does not move on phonation.
E The larynx is totally anaesthetised on the affected side

The larynx is totally anaesthetised on the affected side is incorrect. Only the inferior portion
of the larynx is anaesthetised.
48795

Rate this question:

Next Question

Previous Question Tag Question

Feedback End Session

Difficulty: Difficult

Peer Responses %

Session Progress

Responses Correct: 0

Responses Incorrect: 22

Responses Total: 22

Responses - % Correct: 0%
You are asked to perform a lumbar puncture on a 27-year-old woman who presents with
severe occipital headache and neck stiffness.

A The dural sac in the adult terminates at the lower end of the sacral canal

B The extradural space is composed of a thin layer of avascular connective tissue

C The lumbar puncture can be successfully carried out proximal to L3

D The most common level at which the spinal cord terminates is the disc space
between L1 and L2

E The spinal cord in the average male is 12 inches (30 cm) in length

Explanation 

D The most common level at which the spinal cord terminates is the disc space
between L1 and L2

There is variance regarding where the spinal cord terminates; this can be anywhere between
T12 and L3, but the most common level is between L1 and L2.

A The dural sac in the adult terminates at the lower end of the sacral canal

The dural sac in the adult terminates at the lower end of the sacral canal is incorrect. The
dural sac in adults extends to the level of the second sacral segment.

B The extradural space is composed of a thin layer of avascular connective tissue

The extradural space is composed of a thin layer of avascular connective tissue is incorrect.
The spinal extradural space contains loose fat (which allows the ready diffusion of local
anaesthetic in an extradural block) and the extensive vertebral venous plexus of veins.

C The lumbar puncture can be successfully carried out proximal to L3

The lumbar puncture can be successfully carried out proximal to L3 is incorrect. Given the
cord may terminate as far down as L3, carrying out the lumbar puncture proximal to L3 is not
recommended.
E The spinal cord in the average male is 12 inches (30 cm) in length

The spinal cord in the average male is 12 inches (30 cm) in length is incorrect. The spinal cord
in both adult men and women is 18 inches (45 cm) in length.
48796

Rate this question:

Next Question

Previous Question Tag Question

Feedback End Session

Difficulty: Easy

Peer Responses %

Session Progress

Responses Correct: 0

Responses Incorrect: 23

Responses Total: 23

Responses - % Correct: 0%
A 52-year-old hypertensive man presents with a right hemisphere stroke. Neuroimaging
demonstrates the presence of a recent infarction of the right internal capsule.

A Fibres of the optic radiation lie in the anterior limb

B The anterior limb of the internal capsule lies between the tail of the caudate nucleus
and the lentiform nucleus

C The internal capsule receives its blood supply from the lenticulostriate vessels

D The junction of the anterior and posterior limbs of the internal capsule is termed ‘the
crus’

E The lentiform nucleus itself consists of an outer globus pallidus and an inner
putamen

Explanation 

C The internal capsule receives its blood supply from the lenticulostriate vessels

Infarction of this region is often owing to occlusion of a perforating branch of the middle
cerebral artery, responsible for forming the lenticulostriate vessels.

A Fibres of the optic radiation lie in the anterior limb

Fibres of the optic radiation lie in the anterior limb is incorrect. Fibres of the optic radiation
which run to the visual cortex in the occipital lobe lie in the posterior limb.

B The anterior limb of the internal capsule lies between the tail of the caudate nucleus
and the lentiform nucleus

The anterior limb of the internal capsule lies between the tail of the caudate nucleus and the
lentiform nucleus is incorrect. The anterior limb lies between the head of the caudate nucleus
and the lentiform nucleus.

D The junction of the anterior and posterior limbs of the internal capsule is termed ‘the
crus’

The junction of the anterior and posterior limbs of the internal capsule is termed ‘the crus’ is
incorrect. The junction between the limbs is known as the genu.
E The lentiform nucleus itself consists of an outer globus pallidus and an inner
putamen

The lentiform nucleus itself consists of an outer globus pallidus and an inner putamen is
incorrect. The lentiform nucleus consists of an outer putamen and an inner globus pallidus.
48797

Rate this question:

Next Question

Previous Question Tag Question

Feedback End Session

Difficulty: Difficult

Peer Responses %

Session Progress

Responses Correct: 0

Responses Incorrect: 24

Responses Total: 24

Responses - % Correct: 0%
A 28-year-old man presents with a septic cavernous sinus thrombosis, with high fever, orbital
oedema and proptosis.

A The chin

B The occipital region

C The pinna of the ear

D The skin over the parotid gland

E The upper lip

Explanation 

E The upper lip

The cavernous sinuses lie on either side of the body of the sphenoid and receive venous
blood from the facial veins (via the superior and inferior ophthalmic veins), and the sphenoid
and the middle cerebral veins. The anterior facial vein drains the face and upper lip – hence
the danger of spread of infection from this locus.

A The chin

The chin is incorrect. Venous drainage of the chin is most likely to be via the jugular vein.

B The occipital region

The occipital region is incorrect. Venous drainage of the occipital region is ultimately via the
jugular venous system.

C The pinna of the ear

The pinna of the ear is incorrect. Drainage from the pinna is via the maxillary vein and into
the jugular venous system.

D The skin over the parotid gland


The skin over the parotid gland is incorrect. Similar to the pinna, drainage is via the maxillary
vein, such that the cavernous sinuses are not involved.
48798

Rate this question:

Next Question

Previous Question Tag Question

Feedback End Session

Difficulty: Average

Peer Responses %

Session Progress

Responses Correct: 0

Responses Incorrect: 25

Responses Total: 25

Responses - % Correct: 0%
A cerebral angiogram is performed on a 37-year-old woman following a suspected
aneurysmal bleed.

A The middle cerebral artery courses over the lateral aspect of the temporal lobe of
the cerebrum

B The middle cerebral arteries are the largest arteries that join the circle of Willis

C The middle meningeal artery is an extracranial branch of the internal carotid artery

D The posterior cerebral artery is clearly seen on a lateral carotid angiogram

E The vertebral arteries meet at the foramen magnum to form the basilar artery

Explanation 

B The middle cerebral arteries are the largest arteries that join the circle of Willis

In effect the MCA is the termination of the internal carotid and therefore has a greater
diameter than the other arteries.

A The middle cerebral artery courses over the lateral aspect of the temporal lobe of
the cerebrum

The middle cerebral artery courses over the lateral aspect of the temporal lobe of the
cerebrum is incorrect. The middle cerebral artery passes through the lateral sulcus of the
cerebrum between the temporal and frontal lobes.

C The middle meningeal artery is an extracranial branch of the internal carotid artery

The middle meningeal artery is an extracranial branch of the internal carotid artery is
incorrect. It is a branch of the maxillary artery, which itself is one of the terminal branches of
the external carotid artery. It is particularly vulnerable to injury, which may result in extradural
haemorrhage.

D The posterior cerebral artery is clearly seen on a lateral carotid angiogram

The posterior cerebral artery is clearly seen on a lateral carotid angiogram is incorrect. This is
best visualised by vertebral angiography; in practice, however, MRA is often used to visualise
both the anterior and posterior circulation rather than a traditional angiogram, at least
initially.

E The vertebral arteries meet at the foramen magnum to form the basilar artery

The vertebral arteries meet at the foramen magnum to form the basilar artery is incorrect.
The basilar artery arises from the two vertebral arteries that meet on the under-surface of the
brainstem.
48799

Rate this question:

Next Question

Previous Question Tag Question

Feedback End Session

Difficulty: Difficult

Peer Responses %

Session Progress

Responses Correct: 0

Responses Incorrect: 26

Responses Total: 26

Responses - % Correct: 0%
A hypertensive, heavy- smoking, 73-year-old man suffers a massive cardiac infarct following
occlusion of his anterior interventricular artery (anterior descending artery). Angiography is
performed to demonstrate the coronary vessels.

A The circumflex artery is the major branch of the right coronary artery

B The left anterior descending artery arises above the left posterior aortic cusp

C The left anterior descending artery supplies the majority of the left ventricle

D The posterior interventricular artery is a branch of the circumflex artery

E There is a rich collateral circulation between the right and left coronary arteries

Explanation 

C The left anterior descending artery supplies the majority of the left ventricle

Occlusion of the artery drives presentation with an extensive anterior myocardial infarction.

A The circumflex artery is the major branch of the right coronary artery

The circumflex artery is the major branch of the right coronary artery is incorrect. The
circumflex is a branch of the left coronary artery and supplies the posterolateral left ventricle
and the anterolateral papillary muscle. It also supplies the sinoatrial nodal artery in 38% of
people. The artery supplies 15–25% of the left ventricle in right-dominant systems. If the
coronary anatomy is left-dominant, it supplies 40–50% of the left ventricle.

B The left anterior descending artery arises above the left posterior aortic cusp

The left anterior descending artery arises above the left posterior aortic cusp is incorrect. It is
the left coronary artery that arises above the left aortic cusp. This divides to form the
circumflex and left anterior descending arteries.

D The posterior interventricular artery is a branch of the circumflex artery

The posterior interventricular artery is a branch of the circumflex artery is incorrect. This
arises from the right coronary artery and supplies the posterior one-third of the
interventricular septum.
E There is a rich collateral circulation between the right and left coronary arteries

There is a rich collateral circulation between the right and left coronary arteries is incorrect.
There is no significant collateral circulation.
48800

Rate this question:

Next Question

Previous Question Tag Question

Feedback End Session

Difficulty: Easy

Peer Responses %

Session Progress

Responses Correct: 0

Responses Incorrect: 27

Responses Total: 27

Responses - % Correct: 0%
You are asked to review a chest X-ray requested by a GP for a patient complaining of right-
sided pleuritic chest pain. According to the film it looks like there is consolidation affecting
the right upper lobe, adjacent to the oblique fissure.

A Eighth rib in the mid-axillary line

B Fifth intercostal space

C Tenth rib in the mid-axillary line

D The medial border of the scapula when the arm is fully abducted

E Upper border of the medial third of the clavicle

Explanation 

D The medial border of the scapula when the arm is fully abducted

The oblique fissure does indeed correspond closely to the medial border of the scapula when
the arm is fully abducted.

A Eighth rib in the mid-axillary line

Eighth rib in the mid-axillary line is incorrect. This corresponds to the base of the inferior lobe
of the right lung with the arm abducted.

B Fifth intercostal space

Fifth intercostal space is incorrect. The transverse fissure of the right lung corresponds to the
level of the fourth rib.

C Tenth rib in the mid-axillary line

Tenth rib in the mid-axillary line is incorrect. The lower border of the lung on each side
corresponds to the eighth rib in the mid-axillary line and the tenth rib posteriorly.

E Upper border of the medial third of the clavicle


Upper border of the medial third of the clavicle is incorrect. The medial third of the clavicle is
used to help identify the position of the lung apex, which extends about 4 cm above it.
48801

Rate this question:

Next Question

Previous Question Tag Question

Feedback End Session

Difficulty: Average

Peer Responses %

Session Progress

Responses Correct: 0

Responses Incorrect: 28

Responses Total: 28

Responses - % Correct: 0%
You are asked to assess a 62-year-old man in the clinic for fitness to drive. He suffered an
ischaemic stroke some 3 months ago and has recovered well, but still has a left homonymous
inferior quadrantic visual field defect.

A The left frontal lobe

B The left optic nerve

C The right internal capsule

D The right occipital cortex

E The right parietal lobe

Explanation 

E The right parietal lobe

The optic radiations separate passing through the temporal and parietal lobes. The lower
bundle of fibres (carrying information from the superior visual fields) pass through the
temporal lobe. The upper fibres (carrying information from the inferior visual fields) pass
through the parietal lobe. At these anatomical sites a quadrantanopia is a possible
consequence of localised damage, with an inferior quadrantanopia occurring with parietal
lobe damage.

A The left frontal lobe

The left frontal lobe is incorrect. Frontal lobe damage is associated with mood change and
behavioural disorders rather than visual field deficits.

B The left optic nerve

The left optic nerve is incorrect. Pre-chiasmal lesions result in complete loss of vision from
the affected side.

C The right internal capsule

The right internal capsule is incorrect. Internal capsule lesions do not result in visual field loss;
visual field loss essentially rules out an internal capsule stroke.
D The right occipital cortex

The right occipital cortex is incorrect. Occipital cortex lesions result in a homonymous
hemianopia.
48802

Rate this question:

Next Question

Previous Question Tag Question

Feedback End Session

Difficulty: Easy

Peer Responses %

Session Progress

Responses Correct: 0

Responses Incorrect: 29

Responses Total: 29

Responses - % Correct: 0%
A 28-year-old man, who is a keen bodybuilder, presents with a short history of left upper-
limb discomfort and difficulty in moving his shoulder. On examination he is noticed to have
winging of the left scapula. There is no wasting of the shoulder girdle muscles. With
stabilisation of the scapula, he has a full range of movement and is able to elevate the
shoulder. Sensory testing is normal, as are upper limb reflexes.

A C3,4 nerve root

B C5,6 nerve root

C Diffuse left brachial plexus injury

D Long thoracic nerve

E Spinal accessory nerve (cranial nerve XI)

Explanation 

D Long thoracic nerve

Seven muscles attach the scapula (shoulder blade) to the chest wall and help maintain
normal scapular control:

Trapezius
Levator scapulae
Rhomboids major
Rhomboids minor
Pectoralis minor
Omohyoid
Serratus anterior.

The latissimus dorsi has a small attachment at the base of the scapula but does not
significantly contribute to scapular stability. Of these muscles, the serratus anterior and the
trapezius are the most important. A winging scapula is nearly always associated with partial
or complete paralysis of either serratus anterior or trapezius. Weakness or paralysis of the
serratus anterior, secondary to palsy of the long thoracic nerve, is the most common cause of
winging. The long thoracic nerve (origin C5,6 motor roots, with sometimes a contribution
from C4 ± C7) is thin, fragile and runs an anatomical course in the neck and upper thorax that
makes it susceptible to damage by compression or trauma.
The more common causes of injury include:

Surgery (e.g. radical mastectomy, lymph node biopsy from axilla)


Stretch injury during sports (as in this case)
Viral/post-infectious (brachial neuritis)
Other causes of neuropathy (vascular, toxic, etc.).

A C3,4 nerve root

C3,4 nerve root is incorrect. C3,C4 nerve root lesions are associated with sensory disturbance
over the base of the neck and the tops of the shoulders, with weakness of lateral neck flexion.

B C5,6 nerve root

C5,6 nerve root is incorrect. C5,C6 nerve root damage is associated with weakness of
shoulder abduction, elbow flexion and wrist extension.

C Diffuse left brachial plexus injury

Diffuse left brachial plexus injury is incorrect. Isolated winging of the scapula would not been
seen in this situation.

E Spinal accessory nerve (cranial nerve XI)

Spinal accessory nerve (cranial nerve XI) is incorrect. Accessory nerve damage can also
produce scapular winging via weakness of the trapezius, but this would be milder and would
be expected to be associated with weakness of shoulder elevation, which this patient does
not have.
48803

Rate this question:

Next Question

Previous Question Tag Question

Feedback End Session

Difficulty: Easy

Peer Responses %
Session Progress

Responses Correct: 0

Responses Incorrect: 30

Responses Total: 30

Responses - % Correct: 0%
You are asked to see a 49-year-old alcoholic man who underwent a liver biopsy some 30 min
ago. He has developed right shoulder tip pain, experienced an increase in his pulse of 15 bpm
to 100 bpm, his blood pressure has dropped by 10 mmHg, and is now 105/80 mmHg.

A Axillary nerve

B Intercostobrachial nerve

C Right phrenic nerve

D Right sympathetic trunk

E Right vagus

Explanation 

C Right phrenic nerve

The right phrenic nerve pierces the diaphragm just slightly lateral to the inferior vena caval
foramen. This means that the potential for phrenic nerve stimulation to occur because of liver
capsule inflammation/bleeding is high, and the pain is felt as referred pain to the shoulder.

A Axillary nerve

Axillary nerve is incorrect. Although the axillary nerve carries sensory information from the
shoulder joint, at no time does it have a close anatomical relationship to the liver.

B Intercostobrachial nerve

Intercostobrachial nerve is incorrect. The primary sensory function of this nerve is to relay
information from the skin of the axilla, as such it does not play a role in transmission of pain
related to liver inflammation/bleeding.

D Right sympathetic trunk

Right sympathetic trunk is incorrect. Autonomic changes may result in BP/HR changes but
the increase in heart rate here is likely a normal physiological response to pain or the
procedure. It is also unlikely to be damaged at this site.
E Right vagus

Right vagus is incorrect. Stimulation of the vagus nerve is a rare complication of liver biopsy
and would result in bradycardia +/- fainting. Neither are present here and would also not
explain the shoulder tip pain.
48805

Rate this question:

Next Question

Previous Question Tag Question

Feedback End Session

Difficulty: Easy

Peer Responses %

Session Progress

Responses Correct: 0

Responses Incorrect: 31

Responses Total: 31

Responses - % Correct: 0%
You are reviewing a 52-year-old man who has suffered a myocardial infarction. You suspect
occlusion of the posterior descending coronary artery.

A The anterior left ventricular wall

B The anterior septum

C The posterior portion of the interventricular septum and the posterior left ventricular
wall

D The right atrium

E The right ventricle

Explanation 

C The posterior portion of the interventricular septum and the posterior left ventricular
wall

The right coronary artery arises from the right coronary sinus, giving off branches supplying
the right atrium and right ventricle. The right coronary artery then continues as the posterior
descending coronary artery, which supplies the posterior portion of the interventricular
septum and the posterior left ventricular wall.

A The anterior left ventricular wall

The anterior left ventricular wall is incorrect. This is supplied by the left anterior descending
coronary artery.

B The anterior septum

The anterior septum is incorrect. Like the anterior left ventricular wall, the anterior septum is
supplied by the left anterior descending coronary artery.

D The right atrium

The right atrium is incorrect. The right atrium is supplied by the right coronary artery.

E The right ventricle


The right ventricle is incorrect. The right ventricle is supplied by the right marginal branch of
the right coronary artery.
48806

Rate this question:

Next Question

Previous Question Tag Question

Feedback End Session

Difficulty: Easy

Peer Responses %

Session Progress

Responses Correct: 0

Responses Incorrect: 32

Responses Total: 32

Responses - % Correct: 0%
You are asked to review an 80-year-old woman who has suffered previous neurological
damage. You note that she has a left inferior quadrantanopia on examination.

A Left parietal lobe

B Optic chiasm

C Optic nerve

D Right parietal lobe

E Temporal lobe

Explanation 

D Right parietal lobe

Parietal lobe damage results in a left inferior quadrantanopia.

A Left parietal lobe

Left parietal lobe is incorrect. This would result in a right inferior quadrantanopia.

B Optic chiasm

Optic chiasm is incorrect. Lesions at the chiasm cause bitemporal hemianopia (seen with
pituitary macroadenomas, for example).

C Optic nerve

Optic nerve is incorrect. Pre-chiasmal lesions result in complete loss of one visual field.

E Temporal lobe

Temporal lobe is incorrect. Temporal lobe lesions result in a superior quadrantanopia, rather
than the inferior quadrantanopia seen here.
48807

Rate this question:


Next Question

Previous Question Tag Question

Feedback End Session

Difficulty: Easy

Peer Responses %

Session Progress

Responses Correct: 0

Responses Incorrect: 33

Responses Total: 33

Responses - % Correct: 0%
A 54-year-old woman presents with longstanding tinnitus and evidence of a unilateral fifth
(trigeminal nerve) palsy. Her MRI scan shows evidence of an acoustic neuroma.

A Brainstem

B Cavernous sinus

C Cerebellopontine angle

D Skull base

E Trigeminal ganglion

Explanation 

C Cerebellopontine angle

Lesions at the cerebellopontine angle resulting in fifth nerve damage may include acoustic
neuroma, meningioma and secondary tumour deposits. This patient’s long-standing tinnitus is
a pointer to the acoustic neuroma, confirmed on MRI scan.

A Brainstem

Brainstem is incorrect. Brainstem lesions involving the fifth nerve nuclei may include
brainstem glioma, multiple sclerosis, brainstem infarction or syringobulbia.

B Cavernous sinus

Cavernous sinus is incorrect. Within the cavernous sinus, the trigeminal ganglion may be
compressed by a pituitary tumour extending into the sinus, internal carotid artery aneurysm,
cavernous sinus thrombosis or secondary tumour.

D Skull base

Skull base is incorrect. Acoustic neuromas do not occur at the skull base, and therefore
compression at the skull base cannot account for the symptoms seen here.

E Trigeminal ganglion
Trigeminal ganglion is incorrect. Isolated pathology connected to the trigeminal ganglion is
often related to herpes zoster infection.
48808

Rate this question:

Next Question

Previous Question Tag Question

Feedback End Session

Difficulty: Easy

Peer Responses %

Session Progress

Responses Correct: 0

Responses Incorrect: 34

Responses Total: 34

Responses - % Correct: 0%
A 32-year-old man, 6 feet 7 inches (2.01 m) in height, is admitted via ambulance from
Heathrow airport after a flight from Australia. He is ataxic with double vision and complains
of facial numbness. On examination there is left-sided facial numbness, nystagmus, ataxia and
Horner syndrome. There appears to be some loss of pain and temperature sensation on the
right-hand side.

A Anterior inferior cerebellar artery

B Labyrinthine artery

C Middle cerebral artery

D Posterior communicating artery

E Posterior inferior cerebellar artery

Explanation 

E Posterior inferior cerebellar artery

This is the lateral medullary syndrome (Wallenberg syndrome), which is the most common
brainstem infarction syndrome and is caused by occlusion of the posterior inferior cerebellar
artery. ‘Economy-class syndrome’ as described is more commonly related to deep vein
thrombosis, but brainstem infarction is also a feature of the seat size and pitch in the
economy class cabin. This could potentially result in embolic stroke if a patent foramen ovale
was present and a detailed echo (preferably transoesophageal) would be required as part of
his investigation. In men of this height, Marfan syndrome may be worth considering, which
might contribute to the risk of vertebrobasilar dissection.

A Anterior inferior cerebellar artery

Anterior inferior cerebellar artery is incorrect. AICA occlusion results in lateral pontine
syndrome, the symptoms of which include sudden onset vertigo and vomiting, nystagmus,
dysarthria and falling to the side of the lesion.

B Labyrinthine artery

Labyrinthine artery is incorrect. Occlusion of the labyrinthine artery results in symptoms of


inner ear dysfunction.
C Middle cerebral artery

Middle cerebral artery is incorrect. This results in classical symptoms of anterior territory
stroke, rather than the posterior territory symptoms reported here.

D Posterior communicating artery

Posterior communicating artery is incorrect. Posterior communicating artery occlusion does


not result in the lateral medullary syndrome symptoms seen here, but is a common site for
cerebral aneurysm formation.
48809

Rate this question:

Next Question

Previous Question Tag Question

Feedback End Session

Difficulty: Easy

Peer Responses %

Session Progress

Responses Correct: 0

Responses Incorrect: 35

Responses Total: 35

Responses - % Correct: 0%
Following a rock-climbing accident in which a foothold gave way, leaving him suspended by
one arm, a young man develops weakness of his right hand. He can manoeuvre his arm into
any position but cannot use the hand effectively with weakness of the intrinsic muscles of the
hand, and long extensors of the fingers.

A The C6 nerve root

B The C7 nerve root

C The radial nerve

D The T1 nerve root

E The ulnar nerve

Explanation 

D The T1 nerve root

This patient sustained an injury in a rock-climbing accident that affected the lowest roots
(C8, T1) of the brachial plexus, which provides the motor supply to the intrinsic muscles of
the hand and the long flexors and extensors of the fingers. The deformity caused is known as
Klumpke’s paralysis.

A The C6 nerve root

The C6 nerve root is incorrect. C6 and C7 mediate the movements of the shoulder and elbow.

B The C7 nerve root

The C7 nerve root is incorrect. C7 is also involved in mediating movements of the elbow and
the wrist.

C The radial nerve

The radial nerve is incorrect. The radial nerve supplies a number of muscles on the posterior
aspect of the arm. Weakness therefore impacts on elbow, wrist and finger extension.

E The ulnar nerve


The ulnar nerve is incorrect. A proximal ulnar nerve lesion affects the small muscles of the
hand and wrist flexion but not the positioning of the arm.
48810

Rate this question:

Next Question

Previous Question Tag Question

Feedback End Session

Difficulty: Average

Peer Responses %

Session Progress

Responses Correct: 0

Responses Incorrect: 36

Responses Total: 36

Responses - % Correct: 0%
A 49-year-old man presents with acute back pain following an episode of lifting a heavy
weight.

A Ankle plantar flexion

B Eversion of the foot

C Extension of the great toe

D Inversion of the foot

E Knee extension

Explanation 

C Extension of the great toe

In the lumbar spine nerve roots emerge below their respective vertebrae: thus an L4/5 disc
lesion would be expected to affect the L5 root. Although L5 contributes to hip abduction and
extension, knee flexion and ankle dorsiflexion, weakness in isolated L5 root compression is
often minimal because of the contribution of other roots to these movements. Weakness
tends to be maximal in extension of the toes, particularly the great toe, owing to impairment
of extensor hallucis longus, a branch of the deep peroneal nerve (origin L4, 5 and S1, but
predominantly L5 for extensor hallucis longus), which contributes to ankle dorsiflexion, and is
responsible for dorsiflexion/extension of the great toe.

A Ankle plantar flexion

Ankle plantar flexion is incorrect. Ankle plantar flexion is mediated by S1 and S2.

B Eversion of the foot

Eversion of the foot is incorrect. Eversion of the foot is mediated via the S1 nerve root.

D Inversion of the foot

Inversion of the foot is incorrect. This is mediated by the L4 nerve foot.


E Knee extension

Knee extension is incorrect. This is mediated by the L2, 3 and 4 nerve roots.
48811

Rate this question:

Next Question

Previous Question Tag Question

Feedback End Session

Difficulty: Difficult

Peer Responses %

Session Progress

Responses Correct: 0

Responses Incorrect: 37

Responses Total: 37

Responses - % Correct: 0%
During assessment following a stroke, a middle-aged man undergoes detailed neurological
examination.

A Weakness of ankle plantar flexion

B Weakness of forearm flexion

C Weakness of hip extension

D Weakness of knee flexion

E Weakness of wrist flexion

Explanation 

D Weakness of knee flexion

Typically, in a pyramidal tract lesion, the weakness affects the extensors in the arms and the
flexors in the lower limb. As a result, because of the reduced opposition of the other muscle
groups, the arm tends to be held across the body with the elbow and wrist flexed while the
leg is held in extension with a foot drop.

A Weakness of ankle plantar flexion

Weakness of ankle plantar flexion is incorrect. Weakness of flexors (i.e. dorsiflexion of the
ankle) is usually seen in a pyramidal lesion.

B Weakness of forearm flexion

Weakness of forearm flexion is incorrect. In the upper limb weakness of extensors is usually
seen.

C Weakness of hip extension

Weakness of hip extension is incorrect. Hip flexion weakness is more likely to be seen with a
pyramidal lesion.

E Weakness of wrist flexion


Weakness of wrist flexion is incorrect. Wrist extension weakness is more likely, meaning that
the arm is held in flexion across the chest.
48812

Rate this question:

Next Question

Previous Question Tag Question

Feedback End Session

Difficulty: Difficult

Peer Responses %

Session Progress

Responses Correct: 0

Responses Incorrect: 38

Responses Total: 38

Responses - % Correct: 0%
Following an apparent transient ischaemic attack with transient dizziness, poor co-ordination
and vomiting, a patient is demonstrated to have a left homonymous hemianopia with partial
sparing of central vision.

A Left occipital cortex

B Left optic radiation

C Optic chiasm

D Right occipital cortex

E Right optic radiation

Explanation 

D Right occipital cortex

Where a lesion of the right occipital cortex occurs, vision at the fovea is spared. This is
perhaps because there is such a large representation of the fovea in the cortex (possible
bilateral representation) or possibly due to overlapping blood supply (collateral vascular
supply to the occipital pole from posterior branches of the middle cerebral artery).

A Left occipital cortex

Left occipital cortex is incorrect. A lesion of the left occipital cortex results in a right
homonymous hemianopia.

B Left optic radiation

Left optic radiation is incorrect. Optic radiation lesions result in a homonymous hemianopia
without macular sparing.

C Optic chiasm

Optic chiasm is incorrect. Chiasm lesions result in a bitemporal hemianopia.

E Right optic radiation


Right optic radiation is incorrect. Optic radiation lesions result in a homonymous hemianopia
without macular sparing.
48813

Rate this question:

Next Question

Previous Question Tag Question

Feedback End Session

Difficulty: Average

Peer Responses %

Session Progress

Responses Correct: 0

Responses Incorrect: 39

Responses Total: 39

Responses - % Correct: 0%
A 36-year-old man presents to his GP with a feeling of numbness in his left leg. On
examination, he has decreased position sense and light touch and vibration sensation
affecting his left leg to the upper part of the thigh. No other neurological deficit is
demonstrable.

A Left dorsal column lesion

B Left spinothalamic tract lesion

C Partial section of the spinal cord

D Peripheral polyneuropathy

E Sensory root compression

Explanation 

A Left dorsal column lesion

The dorsal columns carry ipsilateral proprioception, light touch and vibration sensation. Given
this patient’s symptoms, a dorsal column lesion therefore represents the most appropriate
answer here.

B Left spinothalamic tract lesion

Left spinothalamic tract lesion is incorrect. Spinothalamic tract lesions cause contralateral
loss of pain and temperature sensation.

C Partial section of the spinal cord

Partial section of the spinal cord is incorrect. Partial section of the cord tends to cause
contralateral pain and temperature sensation, and ipsilateral loss of the modalities carried in
the dorsal columns; there may also be upper motor neurone weakness.

D Peripheral polyneuropathy

Peripheral polyneuropathy is incorrect. Peripheral polyneuropathy would be unlikely given


the unilateral signs.
E Sensory root compression

Sensory root compression is incorrect. Nerve root compression is commonly associated with
pain, which is clearly absent here.
48814

Rate this question:

Next Question

Previous Question Tag Question

Feedback End Session

Difficulty: Average

Peer Responses %

Session Progress

Responses Correct: 0

Responses Incorrect: 40

Responses Total: 40

Responses - % Correct: 0%
You review a 39-year-old sportsman who complains of knee pain. Arthroscopy reveals
damage to the cartilage.

A Chondrocytes secrete collagen only

B It has a blood supply from small arterioles

C It is avascular

D It is rich in type 1 collagen

E Pressure from normal joint loading accelerates damage to cartilage

Explanation 

C It is avascular

Hyaline cartilage receives its nutrient supply from synovial fluid via a process of diffusion.

A Chondrocytes secrete collagen only

Chondrocytes secrete collagen only is incorrect. Chondrocytes secrete proteoglycans and


collagen, and are embedded in the cartilage. They migrate to the joint surface along with the
matrix that they produce.

B It has a blood supply from small arterioles

It has a blood supply from small arterioles is incorrect. It is avascular, receiving its nutrient
supply via diffusion from synovial fluid.

D It is rich in type 1 collagen

It is rich in type 1 collagen is incorrect. Hyaline cartilage is rich in type II collagen and forms a
meshwork containing proteoglycan molecules that retain water.

E Pressure from normal joint loading accelerates damage to cartilage

Pressure from normal joint loading accelerates damage to cartilage is incorrect. Intermittent
pressure from joint loading is essential to maintain normal cartilage function.
48816
Rate this question:

Next Question

Previous Question Tag Question

Feedback End Session

Difficulty: Average

Peer Responses %

Session Progress

Responses Correct: 0

Responses Incorrect: 41

Responses Total: 41

Responses - % Correct: 0%
You review a 54-year-old man with a history of alcoholism; you are concerned that there may
be evidence of portal hypertension.

A Only the portal vein enters the liver via the porta hepatis

B The caudate lobe of the liver does not have its own branch of the hepatic vein

C The hepatic artery supplies 75% of the total liver blood flow

D The normal portal vein pressure is 8–12 mmHg

E The portal vein supplies 75% of liver blood flow

Explanation 

E The portal vein supplies 75% of liver blood flow

The portal vein drains most of the gastrointestinal (GI) tract and the spleen, and constitutes
75% of liver blood flow. Normal portal pressure is 5–8 mmHg, but blood flow increases after
meals. Portal hypertension is generally considered the diagnosis when pressure exceeds 10–12
mmHg.

A Only the portal vein enters the liver via the porta hepatis

Only the portal vein enters the liver via the porta hepatis is incorrect. Both the hepatic artery
and the hepatic portal vein enter the liver via the porta hepatis.

B The caudate lobe of the liver does not have its own branch of the hepatic vein

The caudate lobe of the liver does not have its own branch of the hepatic vein is incorrect.
The caudate lobe receives an independent blood supply from the hepatic portal vein and
artery, and its branch of the hepatic vein drains directly into the inferior vena cava. This
independent blood supply can drive hypertrophy of the caudate lobe in Budd–Chiari
syndrome.

C The hepatic artery supplies 75% of the total liver blood flow

The hepatic artery supplies 75% of the total liver blood flow is incorrect. The hepatic artery, a
branch of the coeliac axis, supplies 25% of the total liver blood flow, and autoregulation of
the flow ensures constant liver blood supply.
D The normal portal vein pressure is 8–12 mmHg

The normal portal vein pressure is 8–12 mmHg is incorrect. 8 mmHg is the upper end of the
normal range for hepatic portal vein pressure, and levels above 10–12 would normally be
considered consistent with a diagnosis of portal hypertension.
48817

Rate this question:

Next Question

Previous Question Tag Question

Feedback End Session

Difficulty: Average

Peer Responses %

Session Progress

Responses Correct: 0

Responses Incorrect: 42

Responses Total: 42

Responses - % Correct: 0%
You are asked to see a 45-year-old man who is haemodynamically compromised and plan to
insert a right subclavian line. He has a body mass index (BMI) of 38 kg/m 2.

A 0.5 cm under the mid-point of the clavicle and 1 cm laterally

B 1 cm under the mid-point of the clavicle and 0.5 cm laterally

C 1 cm under the mid-point of the clavicle and 1 cm laterally

D 2 cm under the mid-point of the clavicle and 1 cm laterally

E 2.5 cm under the mid-point of the clavicle and 2 cm laterally

Explanation 

D 2 cm under the mid-point of the clavicle and 1 cm laterally

In obese patients, the standard position for right subclavian central venous cannulation is 2
cm under the mid-point of the clavicle and 1 cm laterally. In this case it is the greater BMI
which drives larger spacing from the mid-point of the clavicle before cannulation is
attempted.

A 0.5 cm under the mid-point of the clavicle and 1 cm laterally

0.5 cm under the mid-point of the clavicle and 1 cm laterally is incorrect. This approach will
not achieve adequate clearance under the clavicle.

B 1 cm under the mid-point of the clavicle and 0.5 cm laterally

1 cm under the mid-point of the clavicle and 0.5 cm laterally is incorrect. This is the standard
approach for cannulation in thin individuals.

C 1 cm under the mid-point of the clavicle and 1 cm laterally

1 cm under the mid-point of the clavicle and 1 cm laterally is incorrect. Because of the size of
the patient this increases the risk of hitting the underside of the clavicle rather than
cannulating the vein.

E 2.5 cm under the mid-point of the clavicle and 2 cm laterally


2.5 cm under the mid-point of the clavicle and 2 cm laterally is incorrect. This approach runs
significant risk of perforating the lung and causing a pneumothorax.
48818

Rate this question:

Next Question

Previous Question Tag Question

Feedback End Session

Difficulty: Average

Peer Responses %

Session Progress

Responses Correct: 0

Responses Incorrect: 43

Responses Total: 43

Responses - % Correct: 0%
A 42-year-old man with a history of road traffic collision and injury to his back and neck
presents with global muscle wasting of the left hand.

A C7 nerve root

B Median nerve

C Radial nerve

D T1 nerve root

E Ulnar nerve

Explanation 

D T1 nerve root

Global muscle wasting of the hand indicates damage to both the median and ulnar nerves;
this must include damage to the T1 nerve root, which forms part of the structure of both
nerves.

A C7 nerve root

C7 nerve root is incorrect. The ulnar nerve does not usually carry fibres from C7; the
extensive wasting is likely to involve both median and ulnar nerves, therefore C7
radiculopathy alone is unlikely.

B Median nerve

Median nerve is incorrect. The median nerve supplies the lateral two lumbricals, opponens
pollicis, abductor pollicis brevis and flexor pollicis longus and brevis; the remainder are
served by the ulnar nerve. Isolated wasting of abductor pollicis brevis occurs in association
with median nerve damage from carpal tunnel syndrome.

C Radial nerve

Radial nerve is incorrect. The radial nerve supplies the posterior aspect of the upper limb;
disruption of the nerve leads to weakness of forearm extension and wrist drop.
E Ulnar nerve

Ulnar nerve is incorrect. An ulnar nerve lesion would only lead to partial wasting of hand
muscles, rather than the wider symptoms seen here.
48819

Rate this question:

Next Question

Previous Question Tag Question

Feedback End Session

Difficulty: Average

Peer Responses %

Session Progress

Responses Correct: 0

Responses Incorrect: 44

Responses Total: 44

Responses - % Correct: 0%
A patient has been diagnosed with a fast-growing pituitary adenoma. Magnetic resonance
image (MRI) scanning reveals suprasellar extension.

A Abducens nerve

B Hypothalamus

C Oculomotor nerve

D Optic chiasm

E Third ventricle

Explanation 

D Optic chiasm

The pituitary gland occupies the sella turcica, which is a cup-shaped depression in the
basisphenoid bone. The roof of the sella is formed by the diaphragma sellae, a fold of dura,
which is perforated to allow passage of the pituitary stalk. Above the diaphragma lie the
suprasellar cistern, the optic chiasm and the anterior cerebral arteries. The chiasm is most
likely to be subject to compression by a pituitary tumour.

A Abducens nerve

Abducens nerve is incorrect. The lateral walls of the pituitary fossa are formed by the
cavernous sinuses, which contain the internal carotid arteries and cranial nerves 3, 4, 6 and
the first and second divisions of cranial nerve 5. Upward pressure leading to chiasm
compression/pressure is much more likely than lateral extension.

B Hypothalamus

Hypothalamus is incorrect. Pituitary tumours do not usually lead to hypothalamic


dysfunction, but hypothalamic tumours can lead to posterior pituitary dysfunction.

C Oculomotor nerve

Oculomotor nerve is incorrect. Like the abducens nerve, the oculomotor nerve is much less
likely to be affected by pressure than the optic chiasm.
E Third ventricle

Third ventricle is incorrect. This exists between the two thalami and is therefore not subject
to compression by a pituitary tumour.
48820

Rate this question:

Next Question

Previous Question Tag Question

Feedback End Session

Difficulty: Easy

Peer Responses %

Session Progress

Responses Correct: 0

Responses Incorrect: 45

Responses Total: 45

Responses - % Correct: 0%
You are asked to review an X-ray by one of your juniors. She believes that the patient has an
azygous lobe on the CXR.

A It has its own delineated arterial supply

B It has its own delineated venous drainage

C It is formed when the posterior cardinal artery fails to migrate over the lung apex

D It is found in the right mid-zone

E It is found in the right upper zone

Explanation 

E It is found in the right upper zone

An azygous lobe is seen in about 0.5% of routine chest radiographs and is a normal variant. It
is seen as a ‘reverse comma sign’ behind the medial end of the right clavicle.

A It has its own delineated arterial supply

It has its own delineated arterial supply is incorrect. Azygous lobe is a misnomer, as the
azygous lobe is a congenital variation of the upper lobe and does not have its own arterial
supply.

B It has its own delineated venous drainage

It has its own delineated venous drainage is incorrect. The azygous lobe is not a lobe in its
own right, but part of the upper lobe.

C It is formed when the posterior cardinal artery fails to migrate over the lung apex

It is formed when the posterior cardinal artery fails to migrate over the lung apex is incorrect.
The azygous lobe is formed when the posterior cardinal vein fails to migrate over the apex.

D It is found in the right mid-zone

It is found in the right mid-zone is incorrect. It is found in the right upper zone.
48821
Rate this question:

Next Question

Previous Question Tag Question

Feedback End Session

Difficulty: Average

Peer Responses %

Session Progress

Responses Correct: 0

Responses Incorrect: 46

Responses Total: 46

Responses - % Correct: 0%
A 53-year-old woman comes to the General Medical Clinic for review. She has presented with
proximal myopathy, gradually worsening over the past few months, and dysphagia. CT scan
reveals a mass in the anterior mediastinum.

A Aorta

B Heart

C Oesophagus

D Thymus

E Trachea

Explanation 

D Thymus

The anterior mediastinum is bordered anteriorly by the sternum and posteriorly by the great
vessels. It contains the thymus, lymph nodes, fat and vessels. Disorders of the anterior
mediastinum are generally thymic, thyroid (substernal goitre), teratoma (and other germ cell
tumours) and lymphomas (Hodgkin’s disease, non-Hodgkin’s lymphoma).

A Aorta

Aorta is incorrect. The aorta lies posterior to the anterior mediastinum; as such, it cannot be
responsible for the CT scan appearance seen here.

B Heart

Heart is incorrect. The heart is in line with the great vessels and is therefore posterior to the
anterior mediastinum.

C Oesophagus

Oesophagus is incorrect. Although there are symptoms of dysphagia, they are most likely to
be related to dysmotility rather than any structural oesophageal abnormality. The
oesophagus is also posterior to the anterior mediastinum.
E Trachea

Trachea is incorrect. The trachea lies superior to the anterior mediastinum, and is therefore
not responsible for the changes seen here.
48822

Rate this question:

Next Question

Previous Question Tag Question

Feedback End Session

Difficulty: Easy

Peer Responses %

Session Progress

Responses Correct: 0

Responses Incorrect: 47

Responses Total: 47

Responses - % Correct: 0%
You examine a CT abdomen performed on a patient complaining of left-sided abdominal
pain. There appears to be a mass anterior to and in direct contact with the left kidney; it is
not separated by visceral peritoneum from the kidney.

A Left adrenal gland

B Left psoas muscle

C Spleen

D Splenic flexure

E Tail of the pancreas

Explanation 

E Tail of the pancreas

The pancreas ends with a tail which abuts the spleen and the left kidney, and is therefore the
most likely cause of the CT image seen here.

A Left adrenal gland

Left adrenal gland is incorrect. The left adrenal gland lies superior to the left kidney.

B Left psoas muscle

Left psoas muscle is incorrect. The kidney lies anterior to the psoas muscle; this therefore
cannot be the correct answer.

C Spleen

Spleen is incorrect. The posterior end of the spleen is rounded and is directed upward and
backward; it rests on the upper pole of the left kidney, it is therefore superior to it.

D Splenic flexure

Splenic flexure is incorrect. Although the splenic flexure lies anterior to the kidney,
peritoneum divides the colon from the left kidney as it is retroperitoneal.
48823
Rate this question:

Next Question

Previous Question Tag Question

Feedback End Session

Difficulty: Difficult

Peer Responses %

Session Progress

Responses Correct: 0

Responses Incorrect: 48

Responses Total: 48

Responses - % Correct: 0%
A 25-year-old motorcyclist is involved in a road traffic accident and sustains pelvic trauma.
You are asked to review him as he is recovering on the orthopaedic ward to assess nerve
function in the lower limb.

A Loss of power in the biceps femoris muscle

B Loss of power in the peroneus muscle

C Loss of sensation over the anterior-medial aspect of the thigh

D Preserved knee reflex

E Reduced power on adduction

Explanation 

C Loss of sensation over the anterior-medial aspect of the thigh

The femoral nerve branches off to form both the intermediate cutaneous nerve, and the
medial cutaneous nerve, which supplies sensation to the anterior-medial aspect of the thigh.

A Loss of power in the biceps femoris muscle

Loss of power in the biceps femoris muscle is incorrect. The biceps femoris is innervated by
the common fibular branch of the sciatic nerve.

B Loss of power in the peroneus muscle

Loss of power in the peroneus muscle is incorrect. This is innervated by the superficial
peroneal nerve, a branch of the common peroneal nerve.

D Preserved knee reflex

Preserved knee reflex is incorrect. An absent knee reflex would suggest a femoral nerve
lesion.

E Reduced power on adduction


Reduced power on adduction is incorrect. Hip adductors are primarily not innervated by the
femoral nerve. The pectineus is the only adductor muscle that is innervated by the femoral
nerve. The other adductor muscles are innervated by the obturator nerve with the exception
of a small part of the adductor magnus, which is innervated by the tibial nerve.
48824

Rate this question:

Next Question

Previous Question Tag Question

Feedback End Session

Difficulty: Average

Peer Responses %

Session Progress

Responses Correct: 0

Responses Incorrect: 49

Responses Total: 49

Responses - % Correct: 0%
A 44-year-old man presents with right arm pain and paraesthesia which worsens at night. He
has suffered several years with neck pain, which is increased with movement. On examination
you note weak abductor pollicis brevis.

A C6 radiculopathy

B Carpal tunnel syndrome

C Cervical rib

D Pancoast’s tumour

E Ulnar nerve compression at the elbow

Explanation 

C Cervical rib

Key features here include the prolonged history of pain affecting the neck and shoulder, and
selective weakness of abductor pollicis brevis. This implies involvement of C8 and T1 with
sparing of other nerve roots, and therefore suggests a cervical rib leading to thoracic outlet
obstruction.

A C6 radiculopathy

C6 radiculopathy is incorrect. Abductor pollicis brevis has the nerve roots C8 and T1, which
discounts a radiculopathy involving C6.

B Carpal tunnel syndrome

Carpal tunnel syndrome is incorrect. The history of prolonged neck and shoulder pain, which
is worsened by positional change, is more suggestive of thoracic outlet obstruction than
carpal tunnel.

D Pancoast’s tumour

Pancoast’s tumour is incorrect. The absence of a history of chest disease and the prolonged
history counts against an underlying bronchial carcinoma.
E Ulnar nerve compression at the elbow

Ulnar nerve compression at the elbow is incorrect. The ulnar nerve does not supply motor
function to the abductor pollicis brevis muscle.
48826

Rate this question:

Next Question

Previous Question Tag Question

Feedback End Session

Difficulty: Difficult

Peer Responses %

Session Progress

Responses Correct: 0

Responses Incorrect: 50

Responses Total: 50

Responses - % Correct: 0%
In a patient with numbness in the little finger and palsy of abductor pollicis, which one of the
following gives the most likely site of nerve damage?

A At the brachial plexus

B At the elbow to the ulnar nerve

C At the forearm to the dorsal cutaneous branch of the ulnar nerve

D At the forearm to the musculocutaneous nerve

E At the forearm to the radial nerve

Explanation 

A At the brachial plexus

Numbness of the little finger involves the ulnar nerve (C8–T1). Abductor pollicis palsy may
involve weakness of:

abductor pollicis longus – radial nerve (C7–C8) and/or


abductor pollicis brevis – median nerve (C8–T1).

The nature of symptoms here fits best with a nerve root lesion rather than a specific nerve
injury.

B At the elbow to the ulnar nerve

At the elbow to the ulnar nerve is incorrect. The ulnar nerve is not responsible for motor
supply to the abductor pollicis.

C At the forearm to the dorsal cutaneous branch of the ulnar nerve

At the forearm to the dorsal cutaneous branch of the ulnar nerve is incorrect. Dorsal
cutaneous nerve injury would not explain the motor weakness seen here.

D At the forearm to the musculocutaneous nerve


At the forearm to the musculocutaneous nerve is incorrect. The musculocutaneous nerve
(C5–C6) innervates the biceps and brachioradialis muscles; it is responsible for forearm
flexion and forearm sensation.

E At the forearm to the radial nerve

At the forearm to the radial nerve is incorrect. A radial nerve lesion would not account for the
sensory defect seen here.
48827

Rate this question:

Next Question

Previous Question Tag Question

Feedback End Session

Difficulty: Difficult

Peer Responses %

Session Progress

Responses Correct: 0

Responses Incorrect: 51

Responses Total: 51

Responses - % Correct: 0%
A 31-year-old patient has recently begun a period of structured exercise with a personal
trainer. This involves significant work on forearm musculature. He presents with loss of
sensation in the little finger and weak dorsal interosseous muscles.

A Brachial plexus

B C6 radiculopathy

C Median nerve

D Radial nerve

E Ulnar nerve

Explanation 

E Ulnar nerve

Sensation to the little finger is mediated by the dorsal cutaneous branch of the ulnar nerve.
Motor supply to the dorsal interossei, which abduct the fingers, is mediated by C8–T1
components of the ulnar nerve. The most likely site for ulnar nerve damage is at the elbow.

A Brachial plexus

Brachial plexus is incorrect. Brachial plexus injury would be suggested by symptoms involving
more than one nerve distribution, for example both the median and ulnar, or median and
radial nerves.

B C6 radiculopathy

C6 radiculopathy is incorrect. C6 radiculopathy leads to biceps weakness, weakness of wrist


extension, and loss of sensation over the thumb.

C Median nerve

Median nerve is incorrect. Median nerve injury characteristically leads to weakness of the
thenar eminence.

D Radial nerve
Radial nerve is incorrect. The radial nerve does not supply sensation to the little finger,
meaning this cannot be the diagnosis here.
48828

Rate this question:

Next Question

Previous Question Tag Question

Feedback End Session

Difficulty: Easy

Peer Responses %

Session Progress

Responses Correct: 0

Responses Incorrect: 52

Responses Total: 52

Responses - % Correct: 0%
A 40-year-old lady presents with numbness and tingling in the little finger of her right hand,
and a diagnosis of ulnar neuropathy is made.

A Adductor pollicis

B Extensor digiti minimi

C Flexor pollicis brevis

D Flexor pollicis longus

E Radial two lumbricals

Explanation 

A Adductor pollicis

The sensory symptoms here are typical of ulnar neuropathy with loss of sensation over the
little finger. A number of muscles of the hand are innervated by the ulnar nerve, including the
third palmar interosseous, and the adductor pollicis muscles. The weakness of adductor
pollicis leads to weakness of pinch grip. Ulnar neuropathy commonly occurs because of
entrapment at the elbow owing to either injury or osteoarthritis. Other causes include those
associated with mononeuropathy such as multiple sclerosis or diabetes mellitus. Surgical
intervention is often successful where elbow entrapment has occurred.

B Extensor digiti minimi

Extensor digiti minimi is incorrect. This is supplied by the posterior interosseous nerve, a
continuation of the deep branch of the radial nerve.

C Flexor pollicis brevis

Flexor pollicis brevis is incorrect. Flexor pollicis brevis is supplied by the median nerve.

D Flexor pollicis longus

Flexor pollicis longus is incorrect. Flexor pollicis longus is supplied by the anterior
interosseous nerve, a branch of the median nerve.
E Radial two lumbricals

Radial two lumbricals is incorrect. The radial two lumbricals are innervated by the median
nerve.
48829

Rate this question:

Next Question

Previous Question Tag Question

Feedback End Session

Difficulty: Average

Peer Responses %

Session Progress

Responses Correct: 0

Responses Incorrect: 53

Responses Total: 53

Responses - % Correct: 0%
A 48-year-old woman presents to the clinic complaining that her left foot drags and gets
caught when she tries to climb the steps at home. On examination there is weakness of the
foot and ankle dorsiflexors and she has a so-called steppage gait. She has sensory loss over
the lateral portion of the leg extending from below the knee onto the dorsum of the foot.

A Common peroneal nerve injury

B Femoral nerve injury

C L3 nerve lesion

D L4 nerve lesion

E Tibial nerve injury

Explanation 

A Common peroneal nerve injury

This woman has foot drop, a feature of common peroneal nerve injury. It can be temporary, as
in a ‘dead leg’ because of a blow to the back of the knee, such as that caused by a police
baton, or it can be long-lasting, as in this case.

Causes include:

Mononeuritis (the cause here)


Compartment syndrome
A direct traumatic incident such as a laceration.

Physiotherapy is the mainstay of treatment for foot drop, with the use of aids such as a foot
drop splint to reduce the risk of tripping.

B Femoral nerve injury

Femoral nerve injury is incorrect. The most common features of femoral nerve injury include
quadriceps muscle weakness and wasting, loss of knee jerk on the affected side, and
numbness along the medial side of the thigh and anteromedial side of the calf (the L2–L4
dermatomes).

C L3 nerve lesion
L3 nerve lesion is incorrect. L3 is responsible for innervating hip adductors, meaning that an
L3 nerve lesion would not be the cause of the symptoms seen here.

D L4 nerve lesion

L4 nerve lesion is incorrect. L3/L4 is responsible for innervation of knee extensors, and would
not therefore account for the foot drop seen here.

E Tibial nerve injury

Tibial nerve injury is incorrect. The tibial nerve is rarely injured/affected by mononeuritis, it is
responsible for innervation of (via the medial plantar nerve branch): abductor hallucis, flexor
digitorum brevis, flexor hallucis brevis, first lumbrical; and (via the lateral plantar nerve
branch): quadratus plantae, flexor digiti minimi, adductor hallucis, the interossei, three
lumbricals and abductor digiti minimi.
49689

Rate this question:

Next Question

Previous Question Tag Question

Feedback End Session

Difficulty: Easy

Peer Responses %

Session Progress

Responses Correct: 0

Responses Incorrect: 54

Responses Total: 54

Responses - % Correct: 0%
A 22-year-old golfer presents for review. He has noticed a decrease in his grip strength over
the past few weeks and pain over his elbow. On examination he has pain over his elbow
which is worse on wrist flexion and resisted forearm pronation. There is weakness of the
adductor pollicis muscle and loss of pinch grip as well as loss of sensation over the fourth
and fifth fingers of the hand.

A Brachial plexus injury

B Lateral epicondylitis

C Medial epicondylitis

D Median neuropathy

E Radial neuropathy

Explanation 

C Medial epicondylitis

Medial epicondylitis occurs with increased frequency in golfers and patients who have
excessive use of the elbow joint, such as basketball players. Symptoms include pain over the
medial aspect of the elbow, which worsens on resisted wrist flexion and forearm pronation. In
severe cases there is weakness of muscles of the hand and loss of sensation in the ulnar
nerve distribution.

Medical intervention with physiotherapy and non-steroidal drugs is the mainstay of


treatment. Rarely, surgical release is indicated, but is usually successful.

A Brachial plexus injury

Brachial plexus injury is incorrect. Brachial plexus injury is unlikely given the lack of history of
trauma, pain over the elbow and specific symptoms over the ulnar nerve distribution.

B Lateral epicondylitis

Lateral epicondylitis is incorrect. Lateral epicondylitis is associated with pain over the lateral
portion of the elbow and with movements of forearm extensors.

D Median neuropathy
Median neuropathy is incorrect. Median neuropathy is associated with weakness of the
muscles comprising the thenar eminence, and sensory loss over the palmar aspects of the
medial thumb, the index and middle fingers, and the lateral ring finger.

E Radial neuropathy

Radial neuropathy is incorrect. Radial neuropathy is associated with wrist drop and pain in
the forearm similar to tennis elbow.
49690

Rate this question:

Next Question

Previous Question Tag Question

Feedback End Session

Difficulty: Easy

Peer Responses %

Session Progress

Responses Correct: 0

Responses Incorrect: 55

Responses Total: 55

Responses - % Correct: 0%
A 61-year-old patient who suffered a humeral fracture after falling off a ladder presents for
review after being in a cast for the past 8 weeks. He presents with weakness in the deltoid
and sensory loss over the regimental badge region.

A Axillary nerve injury

B Brachial plexus injury

C Neuralgic amyotrophy

D Radial nerve injury

E Ulnar nerve injury

Explanation 

A Axillary nerve injury

The axillary nerve supplies motor innervation to the deltoid muscle and carries sensory fibres
from the regimental badge area. It is likely that the axillary nerve injury in this patient
occurred during his original fall off a ladder and was not discovered at the time of the initial
injury. Axillary neuropathy may also be caused by direct pressure on the axilla, for instance
from using crutches.

B Brachial plexus injury

Brachial plexus injury is incorrect. Brachial plexus injury is more likely to occur as a
consequence of shoulder injury, and deficits occur according to nerve root anatomy rather
than loss involving a specific nerve distribution as here.

C Neuralgic amyotrophy

Neuralgic amyotrophy is incorrect. Neuralgic amyotrophy is characterised by sudden onset of


severe pain over the shoulder, with restricted shoulder movement; over the course of 2–3
weeks atrophy of the affected muscles is also seen.

D Radial nerve injury

Radial nerve injury is incorrect. Radial nerve injury is associated with wrist drop, rather than
the areas of weakness described here.
E Ulnar nerve injury

Ulnar nerve injury is incorrect. Proximal ulnar nerve injury usually occurs at the elbow, and is
associated with weakness of hand flexion at the wrist.
49691

Rate this question:

Next Question

Previous Question Tag Question

Feedback End Session

Difficulty: Easy

Peer Responses %

Session Progress

Responses Correct: 0

Responses Incorrect: 56

Responses Total: 56

Responses - % Correct: 0%
A 44-year-old woman presents complaining that she has trouble opening jars and holding a
pencil. On examination she has weakness of the adductor pollicis muscle, dorsal and palmar
interossei, and the third and fourth lumbricals. There is also sensory loss over the volar
surface of the hypothenar eminence.

A Anterior interosseous nerve

B Median nerve

C Radial nerve

D C7 nerve root

E Ulnar nerve

Explanation 

E Ulnar nerve

The site of compression of the ulnar nerve is likely to be just proximal to, or within, Guyon’s
canal. Common causes of this type of injury include a large, complex multi-lobulated ganglion
and fracture repair, where trauma/post-operative swelling leads to nerve compression.

A Anterior interosseous nerve

Anterior interosseous nerve is incorrect. The anterior interosseous nerve classically innervates
three muscles, the flexor pollicis longus, pronator quadratus and the radial half of flexor
digitorum profundus.

B Median nerve

Median nerve is incorrect. The median nerve supplies the opponens pollicis, abductor pollicis
brevis and flexor pollicis brevis muscles, but not the adductor pollicis.

C Radial nerve

Radial nerve is incorrect. Radial nerve injury results in wrist drop rather than the features
seen here.
D C7 nerve root

C7 nerve root is incorrect. The C7 nerve root is primarily responsible for nervous control of
wrist flexion/elbow extension.
49692

Rate this question:

Next Question

Previous Question Tag Question

Feedback End Session

Difficulty: Average

Peer Responses %

Session Progress

Responses Correct: 0

Responses Incorrect: 57

Responses Total: 57

Responses - % Correct: 0%
A 62-year-old man presents with lower back pain radiating into the posterior part of the tops
of both legs. He also reports trouble with starting and stopping his stream of urine and
difficulty making it to the toilet when he wants to pass stool. His symptoms have gradually
increased over the past 2 weeks. On examination he has local tenderness to palpation over
the lower back. There is diminished light touch in the perianal region and decreased anal
tone. He has bilateral lower limb weakness with diminished reflexes.

A Cauda equina syndrome

B Conus medullaris lesion

C L2 radiculopathy

D Spinal meningioma

E T10 disc lesion

Explanation 

A Cauda equina syndrome

The clinical picture seen here is typical of cauda equina syndrome with lower back pain and
saddle anaesthesia with bowel and bladder disturbance, caused by compression of nerve
roots below the end of the spinal cord. MRI or CT scanning of the lower spine is the
investigation of choice, and where a cause of compression is identified, such as intervertebral
disc herniation, neurosurgical intervention is of value.

B Conus medullaris lesion

Conus medullaris lesion is incorrect. The gradual presentation and loss of ankle jerks support
a diagnosis of cauda equina syndrome rather than conus medullaris, as shown in the table
below.

Conus medullaris
Cauda equina syndrome
syndrome

Presentation Sudden and bilateral Gradual and unilateral

Knee jerks preserved but


Reflexes Both ankle and knee jerks affected
ankle jerks affected
Radicular
Less severe More severe
pain

Low back
More Less
pain

Numbness tends to be more localised to saddle


Numbness tends to be
area; asymmetrical, may be unilateral; no
Sensory more localised to perianal
sensory dissociation; loss of sensation in
symptoms area; symmetrical and
specific dermatomes in lower extremities with
and signs bilateral; sensory
numbness and paraesthesia; possible numbness
dissociation occurs
in pubic area, including glans penis or clitoris

Typically symmetric,
hyperreflexic distal paresis Asymmetric arreflexic paraplegia that is more
Motor
of lower limbs that is less marked; fasciculations rare; atrophy more
strength
marked; fasciulations common
maybe present

Less frequent; erectile dysfunction that


includes inability to have erection, lack of
Impotence Frequent
sensation in pubic area (including glans penis
or clitoris), and inability to ejaculate

Urinary retention and


atonic anal sphincter cause
overflow urinary
Sphincter Urinary retention; tends to present late in the
incontinence and faecal
dysfunction course of the disease
incontinence; these tend to
present early in the course
of the disease

C L2 radiculopathy

L2 radiculopathy is incorrect. L2/3 supplies the muscles which control hip flexion; weakness
of hip flexion, with one side worse than the other, would usually therefore be expected.

D Spinal meningioma

Spinal meningioma is incorrect. Spinal meningiomas are most commonly seen in females in
middle age and usually have a long duration of symptoms before they present (1–2 years).

E T10 disc lesion

T10 disc lesion is incorrect. T10 compressive lesions present with pain/altered sensation
around the umbilicus.
49693

Rate this question:

Next Question
Previous Question Tag Question

Feedback End Session

Difficulty: Easy

Peer Responses %

Session Progress

Responses Correct: 0

Responses Incorrect: 58

Responses Total: 58

Responses - % Correct: 0%
A 62-year-old man comes to the cardiology clinic for review. He has a history of mitral
stenosis and presents with increased shortness of breath. His blood pressure is 142/108
mmHg. On auscultation there is a loud first heart sound, a mid-diastolic murmur loudest at
the apex and an early diastolic murmur loudest at the left sternal edge. The second heart
sound appears quiet. There are prominent V waves on examination of the jugular venous
pressure. Auscultation of the chest reveals evidence of bibasal inspiratory crackles and he has
peripheral pitting oedema.

A Bibasal inspiratory crackles

B Mid-diastolic murmur at the apex

C Early diastolic murmur at the left sternal edge

D Loud first heart sound

E Quiet second heart sound

Explanation 

C Early diastolic murmur at the left sternal edge

An early diastolic murmur at the left sternal edge is suggestive of pulmonary regurgitation.
There are other signs of pulmonary hypertension on examination, including prominent V
waves, that indicate tricuspid regurgitation and peripheral pitting oedema. Possible
treatments include valvotomy or mitral valve replacement. Fluid can be offloaded from the
left atrium by use of diuretics. The intensity of the second heart sound alters with pulmonary
or aortic valve disease. Early diastolic murmurs at the apex are seen in association with aortic
valve disease.

A Bibasal inspiratory crackles

Bibasal inspiratory crackles is incorrect. Signs of left ventricular failure are supportive of a
diagnosis of mitral valve regurgitation.

B Mid-diastolic murmur at the apex

Mid-diastolic murmur at the apex is incorrect. A mid-diastolic murmur is more suggestive of


mitral stenosis.
D Loud first heart sound

Loud first heart sound is incorrect. Mitral valve prolapse or significant mitral stenosis can
accentuate the first heart sound.

E Quiet second heart sound

Quiet second heart sound is incorrect. This is consistent with calcific aortic stenosis.
49694

Rate this question:

Next Question

Previous Question Tag Question

Feedback End Session

Difficulty: Average

Peer Responses %

Session Progress

Responses Correct: 0

Responses Incorrect: 59

Responses Total: 59

Responses - % Correct: 0%
A 41-year-old man comes to the clinic complaining of pain on the lateral aspect of his elbow
and on resisted dorsiflexion of the wrist. He admits to having been doing a lot of DIY around
the house recently and is a keen tennis player in his spare time. Examination reveals
significant tenderness over the lateral aspect of the elbow.

A Lateral epicondylitis

B Medial epicondylitis

C Median neuropathy

D Radial neuropathy

E Ulnar neuropathy

Explanation 

A Lateral epicondylitis

Lateral epicondylitis or tennis elbow occurs in response to repeated mechanical strain at the
tendon insertion into the lateral epicondyle. Pain extends down the forearm and is worse on
resisted dorsiflexion of the wrist. A number of treatment modalities exist, including
physiotherapy, use of orthotic devices, use of non-steroidal anti-inflammatory drugs
(NSAIDs) or ultimately lateral release via surgery. The condition is generally self-limiting and
resolves between 6 months and 2 years after first onset.

B Medial epicondylitis

Medial epicondylitis is incorrect. Medial epicondylitis is also known as golfer’s elbow and
leads to tendonitis affecting the flexors of the fingers and the pronators of the forearm,
including pronator teres, flexor carpi radialis, flexor carpi ulnaris, flexor digitorum superficialis
and palmaris longus.

C Median neuropathy

Median neuropathy is incorrect. Median neuropathy leads to weakness of the muscles which
comprise the thenar eminence.

D Radial neuropathy
Radial neuropathy is incorrect. Radial neuropathy results in weakness of muscles involved in
wrist extension, leading therefore to wrist drop.

E Ulnar neuropathy

Ulnar neuropathy is incorrect. Ulnar neuropathy does not result from lateral epicondylitis, and
leads to weakness of flexor carpi ulnaris, flexor digitorum profundus and the hypothenar
eminence.
49695

Rate this question:

Next Question

Previous Question Tag Question

Feedback End Session

Difficulty: Easy

Peer Responses %

Session Progress

Responses Correct: 0

Responses Incorrect: 60

Responses Total: 60

Responses - % Correct: 0%
A 45-year-old woman presents with weakness of her thumb and an area of numbness over
the thumb base. She has a history of Type 1 diabetes, which is well controlled with an insulin
pump and frequent home blood glucose monitoring. On examination you confirm weakness
of thumb extension and numbness over the first web space.

A C5 nerve root

B C8 nerve root

C Median nerve

D Radial nerve

E Ulnar nerve

Explanation 

D Radial nerve

Via the posterior interosseous branch, the radial nerve supplies both extensor pollicis brevis
and longus, the superficial branch of the radial nerve supplies the skin over the first web
space. As such, radial nerve is the correct answer here. Patients with Type 1 diabetes are at
increased risk of mononeuritis, which fits with the most likely cause here.

A C5 nerve root

C5 nerve root is incorrect. C5 is responsible for sensory supply to the outer aspect of the
upper arm.

B C8 nerve root

C8 nerve root is incorrect. C8 supplies sensation to the little finger. C8 is responsible for
thumb extension, but the sensory loss does not fit with an isolated C8 lesion.

C Median nerve

Median nerve is incorrect. The median nerve is responsible for motor supply to the thenar
eminence, and sensory supply over part of the palm of the hand.
E Ulnar nerve

Ulnar nerve is incorrect. The ulnar nerve supplies sensation to the fifth digit and the lateral
half of the fourth digit, and the corresponding part of the palm.
49696

Rate this question:

Next Question

Previous Question Tag Question

Feedback End Session

Difficulty: Average

Peer Responses %

Session Progress

Responses Correct: 0

Responses Incorrect: 61

Responses Total: 61

Responses - % Correct: 0%
A 38-year-old woman comes to the clinic. She is finding it increasingly difficult to play the
piano because of problems with proximal interphalangeal (PIP) joint flexion on both hands.
There is a past history of multiple sclerosis (MS), although she has not suffered a flare of her
MS for 2 years, and nil else of note. On examination you confirm deficient PIP joint flexion on
both hands.

A Extensor digitorum

B Flexor carpi ulnaris

C Flexor digitorum longus

D Flexor digitorum superficialis

E Lumbricals

Explanation 

D Flexor digitorum superficialis

FDS is supplied by the median nerve and is responsible for flexion of the fingers at the PIP
joints.

A Extensor digitorum

Extensor digitorum is incorrect. Extensor digitorum extends the medial four digits of the
hand.

B Flexor carpi ulnaris

Flexor carpi ulnaris is incorrect. Flexor carpi ulnaris is responsible for flexion and adduction at
the wrist.

C Flexor digitorum longus

Flexor digitorum longus is incorrect. Flexor digitorum longus is a flexor of the second to fifth
phalanges in the leg. It is supplied by the tibial nerve.

E Lumbricals
Lumbricals is incorrect. The lumbricals are responsible for flexion at the metacarpophalangeal
joints and extension at the interphalangeal joints.
49699

Rate this question:

Next Question

Previous Question Tag Question

Feedback End Session

Difficulty: Easy

Peer Responses %

Session Progress

Responses Correct: 0

Responses Incorrect: 62

Responses Total: 62

Responses - % Correct: 0%
A 72-year-old man is recovering from an episode of cerebral ischaemia. You are asked to see
him by the nurses because he is bumping into furniture around his room on the ward, but
tells you he can see things, including relatives who come to visit him, describing what they
are wearing. When you see him it becomes obvious that he has cortical blindness.

A Anterior cerebral artery

B Anterior communicating artery

C Anterior inferior cerebellar artery

D Basilar artery

E Middle cerebral artery

Explanation 

D Basilar artery

This man has cortical blindness, which may be due to bilateral occipital lobe infarcts. The
occipital lobe is supplied by the posterior cerebral arteries, formed from the basilar artery,
which could have been the site of the stroke leading to the clinical presentation seen here.
Given the site of the stroke, other features including significant other sensory deficits are
likely.

A Anterior cerebral artery

Anterior cerebral artery is incorrect. The frontal region is supplied by the anterior cerebral
artery, and anterior cerebral artery infarction is not associated with visual field loss.

B Anterior communicating artery

Anterior communicating artery is incorrect. This artery bridges the two anterior cerebral
arteries; interruption of blood supply through the anterior communicating artery is therefore
most likely to be associated with frontal lobe deficit.

C Anterior inferior cerebellar artery

Anterior inferior cerebellar artery is incorrect. Anterior inferior cerebellar artery occlusion
leads to nystagmus rather than to visual field loss.
E Middle cerebral artery

Middle cerebral artery is incorrect. Parietal and temporal lobe infarcts are most likely to be
related to a middle cerebral artery stroke, leading to quadrantanopic visual field loss.
49700

Rate this question:

Next Question

Previous Question Tag Question

Feedback End Session

Difficulty: Average

Peer Responses %

Session Progress

Responses Correct: 0

Responses Incorrect: 63

Responses Total: 63

Responses - % Correct: 0%
You review a 48-year-old woman who has undergone resection of a left-sided thyroid nodule.
The surgeon has identified a very high risk of damage to the recurrent laryngeal nerve on the
left-hand side.

A Normal function of the vocal cords

B Paralysis of the cricothyroid muscle on the left

C Paralysis of the interarytenoid muscle on the left

D Paralysis of the posterior cricoarytenoid muscle on the left

E Shortness of breath when lying flat

Explanation 

D Paralysis of the posterior cricoarytenoid muscle on the left

Recurrent laryngeal nerve injury leads to a hoarse voice, because of paralysis of the posterior
cricoarythenoid muscle, which is responsible for opening the vocal cord.

A Normal function of the vocal cords

Normal function of the vocal cords is incorrect. A hoarse voice is seen because of recurrent
laryngeal nerve palsy due to loss of cricoarythenoid function.

B Paralysis of the cricothyroid muscle on the left

Paralysis of the cricothyroid muscle on the left is incorrect. The cricothyroid muscle is not
innervated by the recurrent laryngeal nerve.

C Paralysis of the interarytenoid muscle on the left

Paralysis of the interarytenoid muscle on the left. The interarytenoid muscle is innervated by
both the left and the right recurrent laryngeal nerves.

E Shortness of breath when lying flat


Shortness of breath when lying flat is incorrect. Shortness of breath lying flat is related to
diaphragmatic rather than vocal cord paralysis. The diaphragm is innervated by the phrenic
nerve, which does not pass near the thyroid.
49709

Rate this question:

Next Question

Previous Question Tag Question

Feedback End Session

Difficulty: Difficult

Peer Responses %

Session Progress

Responses Correct: 0

Responses Incorrect: 64

Responses Total: 64

Responses - % Correct: 0%
You are reviewing an 84-year-old man who underwent a phrenic nerve crush procedure for
tuberculosis. You expect diaphragmatic paralysis on the affected side.

A Alternative innervation of the diaphragm will be present via the long thoracic nerve

B Blood supply to the diaphragm is unlikely to be affected on the affected side

C He is more likely to hiccough

D Shortness of breath sitting up is likely

E The diaphragm is usually depressed on the affected side

Explanation 

B Blood supply to the diaphragm is unlikely to be affected on the affected side

The diaphragm is well supplied with blood via the pericardiophrenic artery, musculophrenic
artery and the superior phrenic arteries from above, and from below via the inferior phrenic
arteries.

A Alternative innervation of the diaphragm will be present via the long thoracic nerve

Alternative innervation of the diaphragm will be present via the long thoracic nerve is
incorrect. The diaphragm is innervated by bilateral phrenic nerves.

C He is more likely to hiccough

He is more likely to hiccough is incorrect. Hiccoughing is driven by intact nerve supply to the
diaphragm, meaning patients with phrenic nerve crush should be less likely to hiccough.

D Shortness of breath sitting up is likely

Shortness of breath sitting up is likely is incorrect. Diaphragmatic paralysis leads to shortness


of breath lying down.

E The diaphragm is usually depressed on the affected side


The diaphragm is usually depressed on the affected side is incorrect. The diaphragm is
elevated on the affected side on radiological examination.
49710

Rate this question:

Next Question

Previous Question Tag Question

Feedback End Session

Difficulty: Difficult

Peer Responses %

Session Progress

Responses Correct: 0

Responses Incorrect: 65

Responses Total: 65

Responses - % Correct: 0%
A 24-year-old man, who works as a professional body builder, presents to the Emergency
department with a left groin swelling. On examination you can feel a soft mass on the left-
hand side lateral and inferior to the pubic tubercle, which can be reduced if you press on it
gently. It is non-pulsatile.

A Deep inguinal lymph node

B Dilated femoral vein

C Femoral hernia

D Inguinal hernia

E Superficial inguinal lymph node

Explanation 

C Femoral hernia

The position of this patient’s groin swelling is consistent with a femoral hernia, and his
occupation as a body builder fits with increased risk of developing one. They are at
significant risk of strangulation; early surgical intervention is therefore recommended.

A Deep inguinal lymph node

Deep inguinal lymph node is incorrect. The deep inguinal lymph nodes are located under the
cribriform fascia; enlarged nodes therefore would not present as a mass in the femoral
triangle.

B Dilated femoral vein

Dilated femoral vein is incorrect. Palpable dilatation of the femoral vein would be unusual in
the absence of instrumentation to the vein or evidence of venous thrombosis.

D Inguinal hernia

Inguinal hernia is incorrect. Inguinal hernias occur in the inguinal triangle; the position of this
mass is too inferior to be an inguinal hernia.
E Superficial inguinal lymph node

Superficial inguinal lymph node is incorrect. Although superficial inguinal lymph nodes lie in
the femoral triangle, the fact it can be reduced means it is more likely to be a hernia.
49711

Rate this question:

Next Question

Previous Question Tag Question

Feedback End Session

Difficulty: Difficult

Peer Responses %

Session Progress

Responses Correct: 0

Responses Incorrect: 66

Responses Total: 66

Responses - % Correct: 0%
A 62-year-old man presents to the Emergency Department with lower back pain, which
began whilst he was lifting boxes. He has mild asthma for which he takes a low-dose seretide
inhaler, and hypertension controlled with nifedipine, but no other significant past medical
history. On examination, his BP is 138/82 mmHg, pulse is 85/min and regular and he looks to
be in pain. Neurological examination reveals lack of sensation on the lateral aspect of the
dorsum of the left foot, the bottom of the left foot, and weakness of plantar flexion. Ankle
reflexes are diminished bilaterally.

A S1

B L3

C L4

D L5

E L2

Explanation 

A S1

The history is most consistent with an acute lumbosacral disc prolapse related to the lifting
activity. The pattern of sensory and motor involvement is consistent with a predominant S1
lesion (loss of sensation in the lateral aspect of the calcaneus, loss of plantar flexion and
ankle eversion).

B L3

L3 is incorrect. L3 lesions commonly lead to loss of power of knee extension and loss of
sensation over the medial femoral epicondyle.

C L4

L4 is incorrect. The L4 nerve root innervates sensation over the medial malleolus and motor
function of ankle dorsiflexion.

D L5
L5 is incorrect. L5 lesions lead to loss of power of hallux extension and loss of sensation of
the dorsal aspect of the third metatarsophalangeal joint.

E L2

L2 is incorrect. Injuries of the L2 nerve root may present with weakness of hip flexion and loss
of sensation over the anterior medial thigh.

51118

Rate this question:

End Session

Previous Question Tag Question

Feedback

Difficulty: Average
Peer Responses %

Session Progress

Responses Correct: 0

Responses Incorrect: 67

Responses Total: 67

Responses - % Correct: 0%

You might also like